367

It's one of my real analysis professor's favourite sayings that "being obvious does not imply that it's true".

Now, I know a fair few examples of things that are obviously true and that can be proved to be true (like the Jordan curve theorem).

But what are some theorems (preferably short ones) which, when put into layman's terms, the average person would claim to be true, but, which, actually, are false (i.e. counter-intuitively-false theorems)?

The only ones that spring to my mind are the Monty Hall problem and the divergence of $\sum\limits_{n=1}^{\infty}\frac{1}{n}$ (counter-intuitive for me, at least, since $\frac{1}{n} \to 0$ ).

I suppose, also, that $$\lim\limits_{n \to \infty}\left(1+\frac{1}{n}\right)^n = e=\sum\limits_{n=0}^{\infty}\frac{1}{n!}$$ is not obvious, since one 'expects' that $\left(1+\frac{1}{n}\right)^n \to (1+0)^n=1$.

I'm looking just for theorems and not their (dis)proof -- I'm happy to research that myself.

Thanks!

beep-boop
  • 11,091
  • 10
  • 43
  • 69
  • 8
    related: http://mathoverflow.net/questions/23478/examples-of-common-false-beliefs-in-mathematics – Will Jagy Jun 04 '14 at 18:29
  • 3
    another one: http://mathoverflow.net/questions/35468/widely-accepted-mathematical-results-that-were-later-shown-wrong – Will Jagy Jun 04 '14 at 18:33
  • 73
    This question is hard to answer, because an essential skill for a mathematician is being able to rapidly retune your intuition to match the truth. So once you know that a fact is false, very soon it no longer seems obvious. – Nate Eldredge Jun 04 '14 at 19:53
  • 2
    @NateEldredge I agree, but there's *always* someone who refuses to accept that, for example, $0.999...=1$. – beep-boop Jun 04 '14 at 22:00
  • 2
    Every compact set is Jordan-measurable (false): http://math.stackexchange.com/questions/485336/is-there-a-compact-set-which-is-not-jordan-measurable – DGRasines Jun 05 '14 at 20:58
  • 9
    I strongly disagree that the Jordan Curve theorem is obviously true! I would agree that the Jordan Curve theorem for *piecewise smooth curves* is obviously true; but it's also pretty easy to prove, at least compared to the topological version. (Is it really obvious that the Koch snowflake doesn't have some pathological path from the inside to the outside?) –  Jun 06 '14 at 07:04
  • 1
    Perhaps, for each answer, some readers might want to share comments on how they tuned their intuition to render the subject of the answer obvious (I mean making the falseness obvious). Such comments would be useful as well as the answer itself. For example, let's take Monty Hall problem where the first naive guess turns out to be wrong through a brute force case by case analysis. A tuned intuition could be gained from any explanation that does not use brute force analysis, for example, one could use Bayes rule – Jisang Yoo Jun 06 '14 at 08:34
  • 59
    "Every true statement can be proved" – sinelaw Jun 09 '14 at 17:27
  • 1
    @sinelaw Interesting. Can you find counterexample to such a claim (if it's true and we know it, then it can be proved, but if it's true and we don't know it, then it's not necessarily true)? – beep-boop Jun 09 '14 at 20:11
  • 1
    @alexqwx, it took Godel to *disprove* that...it's his first incompleteness theorem (http://en.wikipedia.org/wiki/G%C3%B6del%27s_incompleteness_theorems) – sinelaw Jun 09 '14 at 21:13
  • 1
    @sinelaw, OK please provide a formal proof of your statement. – Ian Jun 11 '14 at 08:15
  • @sinelaw I think you've missed the conditions on Goedel's theorems. He didn't make a claim about *all* formal systems, and had he done so Goedel would have refuted the completeness theorem for first-order predicate calculus that he had proved earlier (I think Hilbert-Bernays's axiomization of such). – Doug Spoonwood Jun 11 '14 at 11:59
  • 1
    You may be interested in these books: [Counterexamples in Analysis](http://www.amazon.com/dp/0486428753), [Counterexamples in Topology](http://www.amazon.com/dp/048668735X), and [Counterexamples in Probability](http://www.amazon.com/dp/0486499987). – Ziyuan Jul 04 '14 at 23:32
  • Related: [What are some counter-intuitive results in mathematics that involve only finite objects?](http://math.stackexchange.com/q/2040811/121411) – Scott Dec 04 '16 at 08:06
  • I'm voting to close this question as off-topic because enough obvious theorems already – Mikhail Katz Sep 28 '17 at 07:00

70 Answers70

264

Theorem (false):

One can arbitrarily rearrange the terms in a convergent series without changing its value.

Cameron Williams
  • 27,540
  • 4
  • 50
  • 99
Start wearing purple
  • 52,227
  • 12
  • 155
  • 218
  • 1
    Do you have an explicit counterexample to this fact? – Daniel Robert-Nicoud Jun 04 '14 at 18:34
  • 4
    @DanielRobert-Nicoud (Conditionally) convergent series. =) – Pedro Jun 04 '14 at 18:39
  • 20
    The series $\sum \frac{(-1)^i}{i}$ is a counterexample; see [this Wikipedia article](https://en.wikipedia.org/wiki/Riemann_rearrangement_theorem#Changing_the_sum) for a discussion. – MJD Jun 04 '14 at 18:39
  • @MJD Thanks! ${}$ – Daniel Robert-Nicoud Jun 04 '14 at 18:41
  • 2
    @O.L. I know (now) that re-arranging the terms gives an incorrect result; e.g. that $\ln(2)=\frac{1}{2}\ln(2)$, but where is the fallacy in re-arranging the terms? – beep-boop Jun 04 '14 at 18:52
  • 10
    @alexqwx Roughly, that some terms can be pushed 'infinitely far away', so that any two finite partial sums differ by arbitrarily many terms. Imagine e.g. taking the positive numbers and rearranging them as 1, 2, 3, 5, 4, 7, 9, 11, 6, 13, 15, etc. with a growing number of odd terms between each two consecutive even terms; then every number still shows up in the sequence eventually, but that doesn't mean that any concept like a 'ratio' of odd-to-even (e.g. _natural density_) is preserved. – Steven Stadnicki Jun 04 '14 at 18:55
  • 1
    @DanielRobert-Nicoud: since there are counterexamples it is not a fact... –  Jun 04 '14 at 19:01
  • 11
    Huh? One can rearrange the terms in any series, convergent or otherwise. Did you leave something out of the statement of your theorem? – bof Jun 04 '14 at 21:25
  • @MJD That series is not a counterexample to the statement "one can rearrange the terms in a convergent series"; in fact, the Wikipedia article you cited gives several explicit examples of rearrangements of that series. – bof Jun 04 '14 at 21:37
  • 35
    Actually, you **can** rearrange terms in a convergent series when computing its value (or otherwise). You can even rearrange terms without changing the value, with a bit of care. What you cannot, is assume that _any_ rearrangement will preserve convergence, or when it does preserve the value converged to. – Marc van Leeuwen Jun 04 '14 at 22:42
  • 1
    @alexqwx There is no fallacy. What happens is that the usual definition of series is given in terms of a very particular way to obtain values, that is not immune to reordering. – Martin Argerami Jun 04 '14 at 22:56
  • 2
    @MarcvanLeeuwen right, I **can** go around saying that $\sum_{n=0}^\infty n = -\frac{1}{12}$, but for most situations this is not helpful and not true. – DanZimm Jun 05 '14 at 10:06
  • 3
    That sounds quite broken, is that not in contradiction to the commutativity of addition? – kutschkem Jun 05 '14 at 15:30
  • 38
    @kutschkem You can always make finitely many swaps of terms (using commutativity) without changing the value, but you can't make _infinitely_ many swaps; continuity isn't preserved across an infinite number of operations. – Steven Stadnicki Jun 05 '14 at 21:36
  • @StevenStadnicki Ok, i think i see, can you point me somewhere where this is made explicit (in the linked wikipedia article, it is producing a counterexample but doesn't say anything about "continuity isn't preserved across an infinite number of operations". – kutschkem Jun 06 '14 at 15:12
  • 1
    If a series does not converge absolutely, there are subsequences going to $\infty$ and $-\infty$. That things are then not going to invariant under reorderings should be clear from people familiar with Hilbert's hotel. – Michael Greinecker Jun 08 '14 at 09:56
  • 1
    Isn't the problem in how we analyze series, not in harmless transformations? – Aleksandr Dubinsky Jun 09 '14 at 15:55
  • 1
    Why is this answer accepted? It seems that, by the nature of the question, none should be. – OJFord Jun 10 '14 at 15:42
  • 1
    @bof clearly it means "Suppose we have a convergent series with sum S. We can rearrange the terms arbitrarily, and the series still converges to the sum S" – M.M Jun 12 '14 at 03:25
  • @MattMcNabb And my comment clearly means "I know what you meant to say, but you didn't say it." – bof Jun 12 '14 at 04:14
  • 2
    @Ollie Probably it's the one that the OP likes the most. There's no way to make a question ineligible to have an accepted answer. – Ryan Reich Jun 12 '14 at 16:04
  • This one is particular amusing. If you alter this to TRUE and change the rules of math on infinity to allow this change, the fundamental theorem of calculus holds. I can't prove the result is completely consistent but seems to work just fine. This also removes the nasty bite out of the Axiom of Choice (the doubling of the sphere doesn't hold anymore). – Joshua Jun 14 '14 at 02:07
  • 1
    I edited this to say what was actually meant. This bugged me the last time it came up but decided not to fix it. – Cameron Williams Jul 16 '15 at 20:42
  • If this is true then it is so strange that I almost want to redefine convergence to make it otherwise. – mathreadler May 18 '16 at 19:33
  • I learnt this the hard way. –  Jul 25 '16 at 18:59
  • @mathreadler No need to redefine convergence; there's already *absolute convergence*, which is exactly what is needed in order to allow arbitrary rearrangements. –  Feb 24 '18 at 04:36
208

A shape with finite volume must have finite surface area.

DanielV
  • 22,195
  • 5
  • 35
  • 67
  • 54
    This is evidently false. Take a clay cylinder of diameter 1. Roll it in your hands so that its diameter becomes $\frac 12$. The volume is the same, but it is now four times as long as it was before, so it has twice the surface area of the original cylinder. (Half the circumference, but four times the length.) Roll it some more, and the surface area increases again. By making the snake very long and thin, you can increase the surface area to infinity while the volume remains constant. This is not only obvious, it's commonplace. – MJD Jun 04 '14 at 20:31
  • 4
    The familiar example of a sponge illustrates another way that a solid could have a finite volume but a large or potentially infinite surface area. – MJD Jun 04 '14 at 20:32
  • 29
    Those are good examples. I know this wouldn't confuse anyone formally trained like most of the posters here, but I was trying to suggest an example for "layman's terms, the average person" as OP requested. It seemed most of the examples so far would not even be interpretable to an untrained individual. – DanielV Jun 04 '14 at 20:41
  • 135
    @MJD: your cylinder example is nice, but it only shows that the surface area can be arbitrarily large. The OP is pointing out that the surface area can literally be infinite. – Greg Martin Jun 05 '14 at 00:00
  • Yes, but to get it to be actually infinite you follow the same principle and use a snake that is thick at one end but gets thinner as you go out to infinity. Then the only surprising thing is that this infinitely long snake can have a finite volume. But we swallowed *that* oddity a long time ago; I mentioned it elsewhere in this thread along with Zeno's paradox. – MJD Jun 05 '14 at 00:15
  • 1
    That is, you don't roll the whole snake, but rather only the right half at wach step. – MJD Jun 05 '14 at 00:18
  • 2
    @MJD: If you keep expanding the surface area of the clay, won't it reach a point in which the distance between the atoms is great enough causing the clay to disintegrate? If that was the case, you shouldn't be able to expand a clay's surface area infinitely (I dunno, I'm no mathematician). – Zol Tun Kul Jun 05 '14 at 00:56
  • 30
    I dunno either, I'm a physics ignoramus. Are you sure clay is made of atoms? – MJD Jun 05 '14 at 01:11
  • 32
    Mathematically this may be true, but physically impossible, as matter is discrete. The limit would be a row of atoms. – Alex Jun 05 '14 at 04:54
  • 2
    Menger Sponge ! 0 Volume Infinite surface area! – Soham Jun 05 '14 at 09:29
  • 1
    You could set an upper limit if you constrained the shape to be convex. – DanielV Jun 05 '14 at 09:32
  • "A shape with finite volume has a well-defined (finite or infinite) surface area" is less ambitious, but probably false as well (unless the term *shape* has some restricted meaning that I am unaware of, here). – Jeppe Stig Nielsen Jun 05 '14 at 17:33
  • In the same vein, but possibly less widely appreciated, an object of finite mass can fail (mathematically speaking) to have a finite centroid. – Andrew D. Hwang Jun 05 '14 at 23:44
  • 1
    Similarly for 2D, "a figure with finite area must have finite perimeter." A rendering of the Mandelbrot set is proof that it's false: finite area, infinite perimeter. – Brian S Jun 06 '14 at 16:29
  • I am no mathematician, not even close. But couldn't this be proved from V/A ratio? In case of cylinder I think it is h/(h+r). –  Jun 10 '14 at 11:32
  • 1
    No good at maths or physics, but even if we take a purely mathematical cylinder, if you're stretching it as far as infinity to prove that its surface area is thus, does not the volume become infinite at exactly the same instant? – deed02392 Jun 10 '14 at 13:36
  • [Koch Tetrahedron](http://www.math.hmc.edu/funfacts/ffiles/30002.2.shtml). There's your proof. – Anonymous Pi Jun 10 '14 at 13:59
  • 1
    @deed02392 No it doesn't, but this question is exactly why the 'clay counter-example' is not obvious. – jwg Jun 11 '14 at 11:46
  • 4
    I'm sad that my highest rated answer to date is a statement that is completely false. – DanielV Jun 11 '14 at 13:57
  • I still don't understand how this statement is false. Since 'infinity' does not exist and is only a mathematical construct to aid our understanding of nature it seems to me that anything finite is indeed finite. – CramerTV Jun 13 '14 at 23:24
  • @CramerTV http://skeptics.stackexchange.com/a/18145 – DanielV Jun 14 '14 at 02:06
  • 1
    Gabriel's Horn is also an example. Volume of $\pi$ with an infinite surface area. – user3932000 Mar 14 '15 at 17:32
  • 1
    @user3932000: The nice thing about Gabriel’s Horn is that it is such a simple shape, a solid of rotation having, as such, convex cross-sections in appropriate planes. That makes it much more surprising than things like sponges or stretched cylinders. – PJTraill Mar 08 '17 at 18:03
  • This is like arguing that a straight line has an area or a plane has a volume. – samerivertwice Mar 28 '17 at 17:21
184

I wish I'd thought of this yesterday, when the question was fresh, because it's astounding. Suppose $A$ and $B$ are playing the following game: $A$ chooses two different numbers, via some method not known to $B$, writes them on slips of paper, and puts the slips in a hat.

$B$ draws one of the slips at random and examines its number. She then predicts whether it is the larger of the two numbers.

If $B$ simply flips a coin to decide her prediction, she will be correct half the time.

Obviously, there is no method that can do better than the coin flip.

But there is such a method, described in Thomas M. CoverPick the largest numberOpen Problems in Communication and Computation Springer-Verlag, 1987, p152.

which I described briefly here, and in detail here.

MJD
  • 62,206
  • 36
  • 276
  • 489
  • Is there really a consistent concept of a uniform distribution of $\mathbb{N}$ or $\mathbb{R}^{+}$ ? I could understand a distribution like $e^{-x}$ producing a random real number from $0$ to $\infty$, but what about a uniform distribution? – DanielV Jun 05 '14 at 13:16
  • 14
    Who said *anything* about a uniform distribution on $\Bbb N$? Certainly not I. – MJD Jun 05 '14 at 13:17
  • So the distribution that person A uses to choose is left unspecified? – DanielV Jun 05 '14 at 13:17
  • 7
    Yes. That was what I intended by saying that the method was unknown to $B$, which is all that is important here. How could I make this clearer? – MJD Jun 05 '14 at 13:18
  • Sorry, I shouldn't address this here, I'll read your link as well as I can and comment on your other post if I need to as I think that's a better place for discussion. I agree, two lay persons playing this game, if asked "is there a better way than a coin toss" would certainly say no, so +1 on the assumption that your result is correct ^_^ I guess I just have trouble associating mechanical processes to uncomputable concepts (like selecting a random real number). – DanielV Jun 05 '14 at 13:24
  • 1
    This could be made completely computable: you can generate computable numbers to within arbitrary tolerance according to a distribution. – Kevin Arlin Jun 05 '14 at 20:44
  • 7
    Yes, absolutely. It is an *actual* strategy that one can *actually* execute, and *actually* predict correctly with probability greater than $\frac 12$. I was at some pains to explain this in my original post, including for example “methods for doing this are well-studied” and such in hope of persuading the doubtful. – MJD Jun 05 '14 at 20:47
  • 1
    I think this is so far the best answer in this thread. +1 to this answer and your previous answer that explained the paradox. – user1551 Jun 05 '14 at 23:42
  • Person A can very easily choose a strategy that keeps person B's odds of guessing correctly very close to 1/2 (close enough that it isn't worth the effort to use this strategy in practice against a knowledgeable opponent), but the probability is greater than 1/2 nonetheless. +1 to you. – Brilliand Jun 06 '14 at 00:33
  • That is the case. I did describe such a strategy by person A in [my original post](https://math.stackexchange.com/questions/655972/help-rules-of-a-game-which-i-dont-remember-its-details/656426#656426). But I also think it's interesting to notice that A *cannot* guarantee to bound $B$'s probability of success to $\left(\frac12, \frac12+\epsilon\right)$ for any $\epsilon$ less than $\frac12$. – MJD Jun 06 '14 at 00:39
  • @DanielV : "the distribution that person A uses" is a concept that is extraneous to the problem at hand, and if I understand correctly merely mentioning this phrase could be a source of confusion. There is no _distribution_; the claim involves universal quantification over all choices of A. That is, if A picks 1 and 7, this works. If A picks -sqrt(2) and 763.45, this works. And so on. – Prateek Jun 06 '14 at 08:24
  • 10
    I don't think it's obvious that no such strategy exist, in fact I think most laypeople would intuitively apply it (in a heuristic form, i.e. "does that number sound big?" in the sense of "would I have been likely to choose two numbers and find this to be the larger one"). The thing that's obvious, and indeed correct, is that the strategy can easily be subverted by person A, by choosing both numbers very big and close to each other. – leftaroundabout Jun 06 '14 at 10:59
  • 1
    In case you don't believe it: **http://jsfiddle.net/GeB8V/** or console this `function pickPos() { return Math.floor(Math.random() * Number.MAX_SAFE_INTEGER); }; function play() { var y = pickPos(); x = pickPos(); z = pickPos(); return ((x < y && x < z) || (x > y && x > z)) ? true : false }; counter = {wins: 0, losses:0}; i = 100000; while(i--) { counter[play() ? 'wins' : 'losses']++ }; counter` – qwertymk Jun 08 '14 at 03:33
  • @qwertymk This assumes that the person B knows person A's probability distribution. – Tyilo Jun 09 '14 at 20:01
  • @qwertymk It also assumes an upper bound on the numbers being chosen. – DanielV Jun 10 '14 at 10:40
  • @DanielV + Tyilo Those points aside it's still pretty amazing that it works – qwertymk Jun 10 '14 at 12:25
  • 46
    This can be explained **very** simply. B guesses A's strategy. If she's wrong, her odds are 1/2. If she's right, her odds are better than 1/2. As long as her odds of guessing A's strategy are better than 0, her odds of winning are better than 1/2. – Aleksandr Dubinsky Jun 10 '14 at 19:54
  • The additional element is that if we turn this into an iterative game, B needs to change her guess in a way that A can't predict with 100% certainty. In short, since B's prediction of A always has a chance of being right, and A's prediction of B's prediction always has a chance of being wrong, B is always ahead. – Aleksandr Dubinsky Jun 10 '14 at 20:08
  • 28
    @AleksandrDubinsky I think you are wrong. A's actual method is to choose from N(0,1). B guesses that A's method is to choose from N(3,1). B will lose a lot more than 50%. – Matthew Finlay Jun 11 '14 at 06:54
  • 5
    @MatthewFinlay In that case, B will almost always say she picked the smaller one. She still has 50% chance to actually pick the smaller one. – TonioElGringo Jun 11 '14 at 13:48
  • 2
    @AD if A has a continuum of strategies (say, probability distributions) how can B "guess it right" with probability $> 0$? – PA6OTA Jun 11 '14 at 13:57
  • 8
    I may have been wrong. There are some guesses that can ruin B's chances. E.g., A's strategy is to make the high number even, and the low odd. B guesses A's strategy is the reverse and always loses. If we average out the chance of A deciding high=even and him choosing high=odd, B is back to 50/50. In some guessing strategies, being wrong hurts. MJD's proposed guessing strategy is great because it can never be to B's disadvantage. I wonder if it's unique, or if there are others with this property. – Aleksandr Dubinsky Jun 11 '14 at 17:28
  • 2
    By the way @MJD, thanks for posting this; it is one of my favorite probability problems! – 6005 Jun 11 '14 at 18:37
  • 1
    @Goos There is a continuum of real numbers. That doesn't prevent B from guessing the median of A's distribution (which is how I interpret the official solution). E.g., if we assume A's distribution is a small range *anywhere* on the real number line, B still has a non-zero chance of guess a number that falls within it (and win). – Aleksandr Dubinsky Jun 12 '14 at 14:04
  • @Goos Ok, I understand. The proof's trick is that between any two real numbers, there is a rational. It's the rational that can be guessed with non-zero probability. – Aleksandr Dubinsky Jun 12 '14 at 18:40
  • @Goos Are you sure? Guessing a rational is like guessing two integers. If I can't guess a rational, I can't guess an integer. If I can't guess an integer, I can't guess an integer that falls within a range. How is it that I can guess a real within the same range? (Working in reverse: If I can guess a real between two reals, then to guess a rational m/n I just need to guess two reals in ranges (m,m+1) and (n,n+1) and round down.) – Aleksandr Dubinsky Jun 13 '14 at 16:07
  • Please avoid extended discussions in comments. – MJD Jun 14 '14 at 01:38
  • There exists a method of generating the numbers for which no better than 1/2 can work. – Joshua Jun 14 '14 at 02:10
  • 2
    @Joshua That is false. Can you explain your supposed method? – 6005 Jun 17 '14 at 16:27
  • By feeding the cited metric back in we can trivially produce a selection pattern that breaks that metric. Of course there is a counter-pattern to defeat that one. Randomly choosing between L1 and L2 strategies should result in 1/2. – Joshua Jun 17 '14 at 18:06
  • 2
    @Joshua You are mistaken. Even if $X$ knows that $Y$ is picking from a normal distribution, there is nothing $X$ can do to reduce $Y$'s probability of success to $\frac12$. – MJD Jun 17 '14 at 18:21
  • Flipping a coin is a red herring; if I want a strategy where my chance of winning is 1/2, then I can simply always keep whichever card I initially pick (or always switch). To do better than this, I only have to give myself some inducement to prefer to keep a larger number. The simplest improved strategy is to keep a positive number and switch if I get a negative number (although I could as effectively compare to a number other than 0 instead). … – Toby Bartels Jan 12 '19 at 01:34
  • … How well this works depends on A's method of choosing the numbers. If A always picks two positive numbers, then this is equivalent to always keeping my number, so no improvement. Similarly if A always picks two negative numbers. But if A always picks one positive number and one negative number, then this strategy is perfect and I always win. To solve the problem, I need to increase my probability above 1/2 no matter what A does, so this is not a solution yet, but it should be enough to banish the idea that no improvement is possible. … – Toby Bartels Jan 12 '19 at 01:40
  • … To get complete solution, all that I have to do is to choose my cut-off point randomly. Since A might confine the numbers to any limited range, possibly quite large (or small) and narrow, I need a probability distribution that assigns positive probability to any nontrivial interval of numbers. But there are plenty of distributions like that, so pick one, use it to randomly select a cut-off point, and switch if my number is below the cut-off point. And that's the solution described by the OP. – Toby Bartels Jan 12 '19 at 01:45
170

This is elementary compared to most of the other examples, but how about

There are more rational numbers than there are integers.

Alex Ortiz
  • 18,901
  • 2
  • 29
  • 69
Scott
  • 1,015
  • 1
  • 13
  • 27
  • 9
    The beauty lies in it's simplicity. The falseness of this one of those very elementary observations the part of my brain that is driven by common sense still refuses to accept. – aRestless Jun 05 '14 at 23:11
  • My brain in general refuses to accept this. :) Seems like even if you restrict $b$ to $\{1, a+1\}$, $a/b$ gives you twice as many values as $a$. – cHao Jun 05 '14 at 23:29
  • 27
    And there are half as many odd integers than (odd or even) integers. – gnasher729 Jun 06 '14 at 07:30
  • I didn't know that, could you explain what is wrong with the following (fallacious) proof. $\forall (n\in \mathbb{N}) \exists (m_1=n,m_2=\frac{n+1}{2})\subset \mathbb{R}$, which should mean that for every integer, there exist at least two real numbers. I think the problem is in the infinitude of natural numbers, and it is a bit like saying $2\infty > \infty$, which is obviously wrong. Is that the catch? – Jori Jun 07 '14 at 10:45
  • 3
    @Jori: I haven’t studied the proofs and terminology in years, but I think a somewhat simple (simplistic?) way of putting it is that the existence of an injection mapping (not one-to-one) does not preclude the existence of a one-to-one mapping. P.S. Some people might consider 2∞>∞ to be obviously _true_. – Scott Jun 07 '14 at 20:08
  • @Jori: The theorem states *rational* numbers, not *real* numbers. And the classic proof is, afaik, (Cantor's diagonalization)[http://en.wikipedia.org/wiki/Cantor's_diagonal_argument]. You can devise a strategy that enumerates the rational numbers deterministically. This assigns one natural number to each rational number, showing the sets have the same size. – Cornelius Jun 09 '14 at 12:12
  • Oops, misread. Will look into your link. Thanks! Does it count for real numbers as well? – Jori Jun 09 '14 at 13:00
  • @Jori: (1) I should have said “the existence of a non-surjective [injection mapping](http://en.wikipedia.org/wiki/Injective_function) (not a one-to-one correspondence) _between two **transfinite** (i.e., infinite) sets_ does not preclude the existence of a one-to-one correspondence. Obviously, the existence of a non-surjective injection mapping between two _finite_ sets ***does*** preclude the existence of a one-to-one correspondence. (Or is this another “obvious” truth that isn’t really true?) – Scott Jun 09 '14 at 14:43
  • 5
    @Jori: (2) ***No!*** As Cornelius implied in his comment, Cantor’s diagonal argument ([correct link](http://en.wikipedia.org/wiki/Cantor%27s_diagonal_argument)) shows that there are infinite sets (such as the set of real numbers) that cannot be put into one-to-one correspondence with the (infinite) set of natural numbers. There are (infinitely many!) different [_transfinite cardinals_](http://en.wikipedia.org/wiki/Aleph_number). Crudely put, not all infinities are equal. – Scott Jun 09 '14 at 14:43
  • @gnasher729: Surely it is not so much false as meaningless or true to say that there are half as many odd integers as there are integers. The surprising thing is that $2\aleph_0 = \aleph _0$. – PJTraill Mar 08 '17 at 17:50
166

I keep harping on this, because I think it's a spectacular example of something that can be demonstrated to be completely obvious (not only because it seems so, but because it was so widely believed for so long) and yet is completely wrong:

Suppose $\Phi$ is a property that might or might not hold of some object. Then there is a collection $S_\Phi$ of all objects with property $\Phi$.

Many serious and even famous mathematicians went ahead with this intuitively obvious but utterly false principle, whose demolition shook mathematics to its foundations and marks the beginning of modern logic and set theory.

(There are many counterexamples, of which the most well known is $\Phi(x) = $ “$x$ is not a member of collection $x$”. For others, see Is the Russell paradox the only possible contradiction to the axiom schema of comprehension due to Frege (1893)? $\{x:P(x)\}$ and Paradox of General Comprehension in Set Theory, other than Russell's Paradox.)

MJD
  • 62,206
  • 36
  • 276
  • 489
  • 25
    +1 for a great example, but I do think the use of "properties", "objects", and "collections" takes away from the impact by being imprecise. There are definitely consistent set theories where there are collections which are not objects and the statement is true in some sense. Stating it using the word sets and in the context of traditional naive set theory and modern widely-used formalizations of set theory would make this answer better in my opinion. – R.. GitHub STOP HELPING ICE Jun 04 '14 at 22:39
  • 6
    Why is this wrong? – seldon Jun 05 '14 at 06:45
  • 5
    I disagree, I think this works quite fine with undefined terms as long as you have a notion of containment, you don't need anything else. – Stella Biderman Jun 05 '14 at 06:52
  • 1
    Joshua is correct here, and this counterintuitive result applies in a great many related situations. For example, W.V. Quine says “There is no denying that this principle is constantly used, tacitly, when we speak of adjectives as true of things: the adjective ‘red’ is true of a thing if and only if the thing is red, and correspondingly for all adjectives.… It is a hard principle to distrust, and yet… [it] must be abandoned or at least somehow restricted.”. One might similarly think that it would be possible to construct a catalog $C_\Phi$ of all books with some property $\Phi$, but it is not. – MJD Jun 05 '14 at 13:14
  • 1
    @mattecapu There is no collection $S_\Phi$ of all collections which are not members of themselves. – MJD Jun 05 '14 at 13:15
  • 1
    Oh I see your point – seldon Jun 05 '14 at 13:24
  • I've begun to find this formulation of the problem to be misleading. One can always consider a "collection" of objects satisfying a formula, since one can consider them individually and notionally, the ones that do are separated from the ones that don't. The problem is precisely what an "object" is, and which collections of them form a "set". In ZF, they are the same thing, but collections defined by arbitrary formulas may not be; this is different from saying they don't exist: rather, they are not subject to the axioms as "objects". – Ryan Reich Jun 05 '14 at 15:18
  • 8
    I really think that is precisely not the problem. The problem here has nothing to do with ZF, with axioms, or with how we exactly define "objects"; it is a fundamental problem with the intuitive notion of what it means for things to have properties. How do you feel about this formulation: “For any property $\Psi$, one can construct a catalog that lists all the books with property $\Psi$?” And the answer is, that for some properties $\Psi$, you simply cannot. Is that sufficiently concrete and not-set-theoretic? – MJD Jun 05 '14 at 15:22
  • 2
    Wow thank you I have heard of Bertrand's paradox but always thought of it as some contrived example that demonstrated a flaw in the current thinking of the time. Seeing it as a counter example to this really demonstrates just how terrified the mathematical community must of been. – Ben Jun 05 '14 at 15:37
  • 2
    I'm inclined to think that having a catalog of Red things is a non-issue because the definition is not recursive. Can you give a reference to a discussion which demonstrates when this intuition is dangerous? – Polymer Jun 05 '14 at 18:41
  • 4
    @Polymer That is, in fact, one way that people have tried to resolve this issue - the magic word for this sort of restriction (that each set only refers to a 'lower class' of sets) is _type theory_. The catch then becomes that there are a lot of mathematical constructions that are very hard to make 'type-safe'. – Steven Stadnicki Jun 05 '14 at 21:54
  • No, that definition is circular. – MJD Jun 06 '14 at 13:20
  • 2
    @MJD No matter how you say it using vernacular terms, you are implicitly leading me to consider a "collection" as a type of "object", or a "catalog" as a type of "book". That's not necessary, nor obvious; in fact, the claim that it is *true* is exactly the content of the axiom of specification (under the appropriate circumstances). Notionally, the objects satisfying the formula can be considered as a whole, but that whole is not necessarily an object of the sort described by the axioms. (IOW: no, a catalog is not a book, so your reformulation does not persuade me.) – Ryan Reich Jun 06 '14 at 17:17
  • "whose demolition shook mathematics to its foundations and marks the beginning of modern logic and set theory" Where in the history of set theory does this come in? – Michael Greinecker Jun 08 '14 at 10:03
  • 10
    This is [Russell's Paradox](https://en.wikipedia.org/wiki/Russell%27s_paradox), discovered in 1901, which destroyed Frege's *Grundgesetze der Arithmetik* (1903) and prior work. The immediate responses to the paradox include Whitehead and Russell's *Principia Mathematica* (1910) and Zermelo's work on axiomatic set theory starting in 1905, which eventually became ZFC, which dominates set theory to this day. – MJD Jun 08 '14 at 15:11
  • 2
    Does the paradox require that a collection need to count as an object? What if the property is simply "the book is not in a catalog?" You cannot make a catalog of uncataloged books. Simply put, if you let definitions (of properties, or what-not) be sophisticated enough that they're Turing complete (or simply circular), then you can't expect them to always halt or even to know if they'll halt. – Aleksandr Dubinsky Jun 09 '14 at 16:40
  • Would the logic equivalent be "This statement is false"? – JAB Jun 09 '14 at 18:30
  • No, that's not equivalent at all. Why would you think that it would be? – MJD Jun 09 '14 at 18:31
  • 1
    @Jab the "logic equivalent" is $$\text{define } P(Q) := \lnot Q(Q)$$ on the surface it looks non recursive, but what about P(P)... – DanielV Jun 10 '14 at 10:35
  • @MJD Of the failure case, I meant. – JAB Jun 10 '14 at 11:33
  • I don't see what the difference is. Naive set theory assumed that any set that can be defined must exist. It's like saying that any statement that can be constructed must be either true or false. "This statement is false" is neither true nor false. – Aleksandr Dubinsky Jun 10 '14 at 15:15
  • 1
    I thought I saw a statement like that in Russel & Whitehead's *Principia Mathematica*, and assumed that their type system ensured there were no contradictions. – nikie Jun 11 '14 at 06:42
  • 3
    I think "This statement is false" has an unfair degree of notoriety compared with "This statement is true", which causes far more trouble in the real world. – Daniel Earwicker Jun 13 '14 at 09:12
  • @R..: I disagree; the use of non-mathematical language is needed to make it surprising to the _average person_ that it is false. – PJTraill Mar 08 '17 at 18:15
121

In a related mathOverflow thread, Gowers pointed out the following obvious but false claim:

Let $I_1, I_2, \ldots$ be subintervals of $[0,1]$ whose total length is strictly less than 1. Then the union of the $I_i$ cannot contain $\Bbb Q\cap [0,1]$.

(Note that if $\Bbb Q$ is replaced with $\Bbb R$, the claim is true.)

I find the fact that all of $\Bbb Q$ can be covered by an arbitrarily small family of intervals to be one of the most bizarrely counterintuitive in all of mathematics.

MJD
  • 62,206
  • 36
  • 276
  • 489
  • Interesting! Is there an explicit example of such a union or can it only be shown to exist? – Ruben Jun 04 '14 at 19:32
  • 22
    You order the rationals $q_1, q_2, \ldots, q_n, \ldots$ and place an interval of $(\frac \epsilon 2)^n$ around each one, choosing $\epsilon$ to be less than $1$. – dfan Jun 04 '14 at 19:37
  • 23
    Of course the theorem is still true if we have a _finite_ number of intervals; one needs the infinite construction to make this work, which is likely where the gap in intuition lies. – Steven Stadnicki Jun 04 '14 at 19:48
  • @dfan does this work for any e that's less than 1? And by order the rationals, do you mean ordering that rationals that are in [0, 1]? The construction still feels off to me. You phrased it in a way to suggest the order of the rations is arbitrary, but if the first rational was 0.00000001 for e = 0.9, then the interval for it is going to be far outside of [0,1]. What am I missing here? – Cruncher Jun 05 '14 at 17:39
  • As an aside, the above non-theorem introduces an important concept for understanding integration. [link](http://en.wikipedia.org/wiki/Riemann_integral) – Polymer Jun 05 '14 at 17:54
  • 1
    @Cruncher Yes it works for any $\epsilon < 1$ since $\sum_{i=1}^\infty \left(\frac{\epsilon}{2}\right)^n < 1$. And yes he's referring to the rationals in $[0, 1]$ though if you think about the argument, you'll see that's not strictly necessary, he could cover all the rationals with finite length. Finally the statement wasn't that the intervals would be equal to $Q\cap [0, 1]$, but that they contain it. – Polymer Jun 05 '14 at 18:00
  • @cruncher You must first make the rationals into a list, say $\frac11; \frac 12, \frac 21; \frac 13, \frac 22, \frac 31; \frac 14, \frac 23, \frac 32, \frac 41; \ldots $. Then around the $n$th rational in the list, you put an interval of width $\epsilon\cdot 2^{-n}$. That is, if the $n$th rational is $q$, then the $n$th interval is $(q-\epsilon\cdot 2^{-(n+1)}, q+\epsilon\cdot 2^{-(n+1)})$, which certainly includes $q$, and which has length $\epsilon\cdot 2^{-n}$. And you can have $\epsilon$ be as small as you like. And yet each rational is contained in one of these intervals. – MJD Jun 05 '14 at 19:59
  • Does $[a,a]$ count as an interval? Then the $I_i$ can just be an ordering of the rational numbers in $[0,1]$. – 2'5 9'2 Jun 07 '14 at 06:53
  • @alex.jordan: no - the requirement is usually either open intervals, or intervals of positive measure. – Henry Jun 08 '14 at 20:09
  • 1
    @Cruncher You are right, that's an issue because it does say "subitntervals" in the answer. So two things: 1) I believe that contstruction can be fixed so that our intervals go outside of $[0, 1]$; for example (this needs a little checking though, it could be impossible to do that) if some of your sub-intervals lie outside $[0, 1]$, just keep shrinking $\epsilon$ until every sub-interval is in $[0, 1]$. 2) Even if you can't do that, it's still amazing that you can cover the rationals with intervals whose sum is arbritrarily small, wether you can fit all the sub-intervals into $[0, 1]$ or not. – Ovi Dec 20 '18 at 13:44
  • @Cruncher It tells us that the rationals are really negligible when you're trying to measure length; for example, you can say that the length of $[0, 1] - \mathbb{Q} = 1$. If you are more interested, the thing to google for is "Lebesgue measure". The rationals have Lebesgue measure $0$, and this is the standard way to show it. – Ovi Dec 20 '18 at 13:46
  • @Cruncher Numberphile has an interesting video on it https://www.youtube.com/watch?v=aIggWlKr41w – Ovi Dec 20 '18 at 13:47
  • @Polymer : Right, the concept of a set of measure zero. – Toby Bartels Jan 12 '19 at 02:47
  • 1
    @Ovi : The subintervals don't need to be centred on the rationals, so you don't need to shrink ε, just take the intersection with [0,1]. – Toby Bartels Jan 12 '19 at 03:19
  • This is not an arbitrarily small family of intervals, but a family of intervals with arbitrarily small total measure. – Rosie F Jul 16 '20 at 17:03
  • This problem was discussed in a 3Blue1Brown video about music theory – Some Guy Jan 29 '22 at 01:29
112

If a function $f(x)$ has an horizontal asymptote, then $\lim f'(x) = 0$

Ant
  • 20,466
  • 5
  • 41
  • 97
  • 23
    +1 Love this one! A counterexample is $\frac{\sin{x^2}}{x}$. – Gamma Function Jun 05 '14 at 10:30
  • 6
    A sufficient condition for this to be true is for the second derivative to be bounded. – Alex Provost Jun 05 '14 at 12:38
  • 6
    @A.P. or even just that $\lim f'(x)$ exists. – Ant Jun 05 '14 at 17:50
  • 26
    As an undergraduate I attempted to write an $\varepsilon$-$\delta$ proof that if $f'(a)>0$ then there is some open interval containing $a$ on which $f$ is increasing. And I thought my ability to write $\varepsilon$-$\delta$ proofs was missing something because I couldn't figure out how to do it. – Michael Hardy Jun 05 '14 at 20:41
  • 4
    Ah, we have almost-duplicate responses. The interesting thing is that you can add the following restrictions to make it even more bizarre: $f$ is $C^\infty$, bounded, and monotonic. – Kaj Hansen Jun 06 '14 at 09:28
  • @Kaj Hansen: Can you show me an example $f$? :) – The_Sympathizer Jun 07 '14 at 00:07
  • Though I think I can imagine it, actually: think of an $f$ that has smooth, but ever-steeper, and ever-shorter-height, stairsteps when graphed. Then the $\lim$ does not exist. (Why?) But what about one where the $\lim$ exists? – The_Sympathizer Jun 07 '14 at 00:08
  • I think what you describe can actually work with some tinkering. See my 10-upvote response below. – Kaj Hansen Jun 07 '14 at 00:13
  • @mike4ty4 If the limit exists, then it must equal 0. – Gamma Function Jun 09 '14 at 19:54
  • Is this statement still true if $\text{f} \left( x \right)$ is not allowed to cross its own horizontal asymptote? Because I think that is where the false intuition might be coming from. – Ryan Jun 10 '14 at 15:08
  • 1
    @Ryan You mean if $f(x)$ is monotonic? Yes, even it this case we can't say that the limit equals $0$, as others have pointed out – Ant Jun 10 '14 at 15:58
  • A widespread definition among illiterates is that an asymptote is a line that the graph of a function keeps getting closer to but never touches. – Michael Hardy Jun 13 '14 at 18:10
  • @Ryan If you take $f(x)=\frac{\sin(x^2)+2}{x}$, then $f(x)$ is eventually positive, but the derivative still doesn't have a limit. – Wojowu Apr 03 '16 at 09:15
  • @MichaelHardy re "illiterates": Are you objecting to the "never touches" part, or the vagueness of the definition? If the former, then that's a bit harsh given that's literally (ha!) the etymology of the word. – A. Rex Nov 18 '16 at 18:38
  • Primarily the "never touches" part: They will tell you that if a curve crosses or touches a line, then that line is by definition not an asymptote of the curve. But also, the function $x\mapsto y=\dfrac{\sin x} x$ has $y=0$ as an asymptote, but not only does it keep returning to cross that line, but it also does not keep getting closer, even "eventually". $\qquad$ – Michael Hardy Nov 18 '16 at 19:24
  • 2
    Alas, I have constantly used this "fact" to find equilibrium solutions to differential equations. Now all of my work is in jeopardy. – Display name May 18 '18 at 17:38
105

$ 0.\overline{9} < 1 $

Probably the most famous of the "obvious" but false.

MichaelRushton
  • 121
  • 1
  • 1
  • 4
  • I wonder how "obvious" this is among people who know how the real numbers are actually defined. – Tanner Swett Jun 09 '14 at 04:27
  • 9
    The question did ask what the *average* person would consider true. – MichaelRushton Jun 09 '14 at 07:56
  • Another way to state this is "every real number can be written as an infinitely long decimal expansion in exactly one way.", to make it more obvious to most people. – RemcoGerlich Jun 09 '14 at 14:46
  • 20
    This is by far the best one on the page imo. Some of the others are so complex to the average person that they wouldn't be "obviously" true to almost anyone. – Sam Creamer Jun 10 '14 at 20:20
  • 2
    In real numbers it is false, but it is true for the hyperreals. It's only by convention that we default to using the theory of real numbers instead of some other system. – M.M Jun 12 '14 at 03:31
  • 4
    Will not considering any type of numbers 0.999... straight up is a number but this "obvious" is actually true, let me explain. 0.999... is equal to 1 - 1/∞. Which makes a weird type of infinite which is infitly close to one. – Binary Freak Jul 23 '14 at 20:06
  • But 0.999... isn't equal to 1 - 1/∞ (unless by 1 - 1/∞ you just mean 0). – MichaelRushton Jan 19 '15 at 20:49
  • 11
    I believed this until I was $9.\overline9$ years old. On my $10\,\text{th}$ birthday, I saw the light.    :-)    ⁠ – Scott Dec 04 '16 at 08:34
  • Remember that an infinite string of $0$s that ends with a $1$ never ends in a $1$, because that'd contradict the idea that infinity is never-ending – Some Guy Jan 29 '22 at 01:31
101

The falseness of

Let $S$ be an infinite family of strictly positive numbers. Then $\sum S = \infty$

has been boggling people for thousands of years. It is the basis for Zeno's paradox, but if you think Zeno's paradox is old and tired, consider that it is also the basis for the Gabriel's Horn paradox (also mentioned in this thread), which still puzzles people.

MJD
  • 62,206
  • 36
  • 276
  • 489
  • 1
    This is, in my opinion, the best example here. It's easily understood by a layman with a little explanation, and at first glance, it would seem *incredibly* obvious that if you keep adding up numbers, it would always get bigger. – jpmc26 Jun 04 '14 at 23:45
  • 7
    @jpmc26 It _does_ always get bigger as you keep adding numbers; it just doesn't necessarily do so without limit. – Mike Scott Jun 05 '14 at 07:26
  • 5
    On the other hand if the family is uncountable, it holds. – user87690 Jun 05 '14 at 13:27
  • 2
    I just read this one and the comments... and it's really freaky. (I have a master's in physics, so I'm not entirely unfamiliar with these sorts of things.) – Almo Jun 05 '14 at 17:31
  • 25
    I feel like this isn't that crazy, though maybe that's because I have experience, but I mean $1.1111\cdots=1+0.1+0.01+\cdots$. – JLA Jun 05 '14 at 21:10
  • @Almo You most likely have studied these in you basic calculus courses. It's just an infinite sum, and some infinite sums converge to a value instead of diverging. Remember all those series convergence/divergence tests around the time you first studied integrals? – jpmc26 Jun 05 '14 at 22:06
  • 15
    Ooooh! I read it as "strictly positive *integers*" which I believe would diverge. Right? (Too much programming) – Almo Jun 05 '14 at 22:30
  • 2
    @Almo The sum of an infinite series of positive integers necessarily diverges to infinity, yes (it can't be less than the sum of an infinite series of 1s). – Brilliand Jun 05 '14 at 22:56
  • 2
    @user87690 What if $\forall s \in S, \forall x \in \mathbb{R}^{+} \setminus 0: s < x$; that is, $s$ is infinitesimal? – wchargin Jun 06 '14 at 02:33
  • 1
    @WChargin: Then I guess it doesn't hold again since finite sum of infinitesimals is infinitesimal so any finite subsum is less than say $1$. For my previous comment I assumed that strictly positive number is real number. – user87690 Jun 06 '14 at 07:20
  • 1
    @user87690 I don't think that is right. The sum of an uncountable number of positive infinitesimals will not converge, by the transfer principle (since it doesn't converge for any positive real number). Alternatively, it doesn't converge because it will eventually sum to more than $1$ after you reach a hyperinteger number of terms (still less than $\omega$). – Mario Carneiro Jun 10 '14 at 17:56
  • @MarioCarneiro I don't say it converges, I just say it isn't ∞. To be clear, the sum I'm talking about is defined as limit of the net of finite subsums. Isn't it true that any finite sum of infinitesimals is infinitesimal by ordinary induction? – user87690 Jun 10 '14 at 19:50
  • @user87690 I may be mistaken, but I believe this is false, in the sense that if you try to write down what you actually mean by "finite", you will either explicitly reference nonstandard numbers or else will be forced to allow nonstandard natural numbers to also be "finite". For simplicity, consider instead a countable sum of equal numbers (the uncountable sum reduces to this case). For any positive number $\epsilon$ there is a number of terms $N$ such that the sum is greater than $1$ (and by multiplying this by $M$ you can make it as large as you like). (cont) – Mario Carneiro Jun 10 '14 at 23:24
  • ... By the transfer principle this is also true for infinitesimal $\epsilon$, although now $N$ is also a nonstandard natural number (but still "finite"). In other words, if you add one gazillionth to itself a gazillion times, you still get one. If you accept the existence of infinitesimal numbers, you are forced to also accept hyperintegers, and so the sum still diverges to $\infty$. – Mario Carneiro Jun 10 '14 at 23:28
  • The statement you are trying to make is that a sum of $n$ infinitesimal numbers where $n$ is *standard* is also infinitesimal, but it is not possible to state this within the model. (Alternatively, you could say that the sum of 2, and 3, and 4 infinitesimals is infinitesimal, but not for general $n$, since it is also true that for any infinitesimal $\epsilon$ there is an $n$ such that $n\epsilon>1$.) – Mario Carneiro Jun 10 '14 at 23:38
  • @user87690 There are finite integrals $\int_1^\infty y, y>0$ so I'd expect an uncountable number of positive values could still have a finite sum. – Mark Hurd Jun 11 '14 at 05:03
  • 1
    @MarkHurd $S_n := \{x ∈ S: x > 1/n\}$. If some $S_n$ is infinite, then clearly $∑S = ∞$. On the other hand if all of them are finite, then $S$ as their union is countable. – user87690 Jun 12 '14 at 07:10
  • @MarioCarneiro I understand, you are right. But I wasn't trying to state anything within the model. We have just objects $ω$, $\mathbb{R}$ with elementary embedding to some $\mathbb{R}^*$. Infinitesimals are certain members of $\mathbb{R}^*$, we are summing in $\mathbb{R}^*$ and infinite sum is a limit of net of finite subsums where finite means bijective to a member of $ω$. – user87690 Jun 12 '14 at 07:18
  • Zeno is associated with 9 surviving to 40 attributed arguments. All references. It's error-prone to associate him with a variant of what this answer states. However anyone interested in validating such asserted association may find article on [mathpages.com / Kevin Brown's Reflections on Relativity](http://www.mathpages.com/rr/s3-07/3-07.htm) insightful. It attempts to present what's actually known about Zeno's work and show that the 2 most famous paradoxes "*far from being vitiated by the convergence of infinite series, they actually depend on the convergence of the geometric series*". – n611x007 Jun 12 '14 at 12:36
  • @user87690 Accepted, and the fault with my statement is the level sets (which is what your $S_n$ are) of an integral are _all_ infinite, but all infinitesimals. – Mark Hurd Jun 12 '14 at 16:11
  • It's interesting that $\sum S = \infty$ and $\sum 1/n = c;finite$ are both on this list. So how about "if something is obviously true it can't be obviously false" as $\sum positive = \infty$ clearly is both. – fleablood Apr 03 '16 at 23:03
69

Every chain of subsets of $\mathbb N$ is countable.

bof
  • 70,356
  • 5
  • 80
  • 153
  • 2
    So far I find this to be the most true for me. For the life of me I still cannot give myself a good metaphor that make it obviously false, even though I know mathematical counterexample. Other answers, either it never found them obvious, or if I did, I can quickly fix the misconception by an alternative way to look at it. You probably should add in counterexample for this one in the answer. – Gina Jun 09 '14 at 02:30
  • 4
    By "chain", do we mean a collection of subsets of N that is totally ordered by the subset relation? You can represent the interval $[0, 1]$ as such a chain. For each number $x$ in that interval, the subset contains (rounding down) the first $9x$ one-digit numbers, the first $90x$ two-digit numbers, the first $900x$ three-digit numbers, and so on. – Tanner Swett Jun 09 '14 at 04:15
  • @TannerSwett That is the meaning, and yours is a good example. – bof Jun 09 '14 at 05:01
  • 14
    @TannerSwett More straightforwardly, consider Dedekind cuts of rational numbers. – Dustan Levenstein Jun 12 '14 at 03:56
  • 5
    @DustanLevenstein Sure. Or, let $\mathbb Z[i]$ be the set of all Gaussian integers, and let $S_\alpha=\{z\in\mathbb Z[i]:0\lt arg(z)\lt\alpha\}$. – bof Jun 12 '14 at 04:12
  • The proof appeared in some posts on this site, for example [here](http://math.stackexchange.com/questions/253192/chain-of-length-2-aleph-0in-p-mathbbn-subseteq) and [here](http://math.stackexchange.com/questions/322279/examples-of-transfer-via-bijection/322282#322282). – Martin Sleziak Jun 12 '14 at 06:15
68

Keller's conjecture is obviously true:

Let $\Bbb R^n$ be completely covered with identical, non-overlapping $n$-cubes. There must be two cubes that share a face.

(For example, when $n=2$ we cover the plane with little square tiles, and the conjecture states that there must be two tiles that share an edge. This is true.)

However, the conjecture is false for all $n>7$.

MJD
  • 62,206
  • 36
  • 276
  • 489
  • 96
    I'm not sure I buy this example myself, because I'm not sure anything concerning 7-dimensional anythings can be considered intuitively obvious. – MJD Jun 04 '14 at 18:55
  • 7
    Wouldn't the fact that it's true for $n = 1, 2, 3$ (and $n = 4, 5, 6$) lead you to suspect that it might be true for all $n$? – Jesse Madnick Jun 05 '14 at 04:47
  • 13
    What does "face" even mean in 7 dimensions? Is it a 6-dimensional thing or a 2-dimensional thing? – user541686 Jun 05 '14 at 07:04
  • 11
    What precisely do we mean by "share a face" in this context? – goblin GONE Jun 05 '14 at 10:20
  • 4
    It's 6-dimensional: the hypercube is $[0,1]^7$ and you get a face by fixing one coordinate to be $0$ or $1$. – Kevin Arlin Jun 05 '14 at 10:43
  • 7
    what do you mean by "tiling"? – electronp Jun 05 '14 at 12:04
  • 2
    please give a reference for this fact. – electronp Jun 05 '14 at 12:04
  • 1
    Without further qualification, I would assume that tiling meant on a n-grid, which I'm sure means that the conjecture IS trivially true. Perhaps that's obviously not what's meant to geometers, but I think other non-geometers like me could make the same mistake. I think it would be helpful to mention what is really meant by a "tiling". – Jim Oldfield Jun 05 '14 at 13:00
  • Why would you think that? That's not what it means in general, where there are all sorts of tilings that are not square grids. Tiling in mathematics just means that you cover the plane (or whatever) with the tiles so that they do not overlap, except at their boundaries. – MJD Jun 05 '14 at 13:10
  • 4
    @Jim, for example $R^3$ could be covered by planes in a regular pattern, rotated against each other. I can't even imagine imagining how this would work in $R^8$. – gnasher729 Jun 06 '14 at 07:28
  • @gnasher729: All I meant was that the language used in the statement was confusing, but MJD's now edited their answer so it's clearer. – Jim Oldfield Jun 06 '14 at 11:14
  • What about higher-dimensional "edges"? (e.g. for $n=8$, there are no shared $(n-1)$-cubes, but what about $(n-2)$-cubes and so forth?) Or does the failure of the conjecture for $n>7$ imply that edges need not be shared either? – JAB Jun 09 '14 at 18:50
  • 2
    @gnasher729 I need to start living in higher reality planes. – Anonymous Pi Jun 10 '14 at 13:57
  • 3
    This topic might make a nice contribution to [how does intuition fail for higher dimensions?](http://math.stackexchange.com/q/258358) as well. – MvG Jun 15 '14 at 22:01
65
  • If $U$ is an open subset of $\mathbb{R}^n$ that is homeomorphic to $\mathbb{R}^n$, one might think it "obvious" that it's in fact diffeomorphic to $\mathbb{R}^n$ (perhaps thinking something like "topologically it looks like $\mathbb{R}^n$, and differentiably it's locally trivial"). In fact, this is true (but by no means obvious!) for $n\neq 4$. But for $n=4$ it is false: there exist exotic $\mathbb{R}^4$'s (differentiable manifolds that are homeomorphic, but not diffeomorphic, to $\mathbb{R}^4$), including "small" ones which are diffeomorphic to an open subset of $\mathbb{R}^4$.

  • Much less profound, but still fun: it's "obvious" that the sum of two convex open sets in the plane whose border is $C^\infty$ also has a $C^\infty$ border (perhaps thinking something like "the border of the sum is parametrized by a smooth function of the borders of the summands"). But this is false: in fact, the border of the sum is always $C^{20/3}$ (meaning six times differentiable and with a sixth derivative which is appropriately Hölder) and no more in general. A simple counterexample is given by the epigraphs of $x^4/4$ and $x^6/6$. For details, see Kiselman, "Smoothness of Vector Sums of Plane Convex Sets", Math. Scand. 60 (1987), 239–252.

Gro-Tsen
  • 4,989
  • 19
  • 41
64

$i^i$ is imaginary.$\ \ \ \ \ \ $

bof
  • 70,356
  • 5
  • 80
  • 153
  • I remember finding it amazing the first time I did this calculation . – Tim Seguine Jun 05 '14 at 20:15
  • 63
    Believing $i^i$ is well-defined is illusory. – Marc van Leeuwen Jun 06 '14 at 13:26
  • 6
    I'm certainly not saying anything profound. One simply should not write down things that look like constant expressions but aren't. – Marc van Leeuwen Jun 06 '14 at 13:40
  • 2
    $i^i\approx 0.207879$ – Anonymous Pi Jun 06 '14 at 14:24
  • 1
    @bof: Well to be "obviously true" for me requires obviously making sense. Anyway I wouldn't consider this one any more "obviously true" than "$i\times i$ is imaginary". – Marc van Leeuwen Jun 06 '14 at 16:02
  • 17
    None of the infinitely many values for $i^i$ are imaginary :) – The_Sympathizer Jun 07 '14 at 00:10
  • 5
    @bof: Yes. And to say that "$i^i$ is imaginary" is false _even when taking the multivalued nature into account_. That's what I meant. – The_Sympathizer Jun 07 '14 at 03:04
  • 2
    @xfix Um, no it's not. – apnorton Jun 09 '14 at 02:56
  • 7
    @anorton Depends on how you define "imaginary". The usual definition is "having real part 0". and 0 certainly has real part 0. – Snowbody Jun 09 '14 at 16:50
  • 3
    @Snowbody Ah. I guess I have seen that definition fairly often. I typically think of "imaginary" as in the set $\mathbb{C}\setminus \mathbb{R}$ (Hence my confusion) – apnorton Jun 09 '14 at 16:52
  • 4
    @Snowbody The usual definition of "imaginary" is "having nonzero imaginary part". The definition of "purely imaginary" is "having real part zero". – bof Jun 09 '14 at 20:47
  • 2
    @AnonymousPi $i = \exp\left(\frac{(4n + 1) \pi i}{2}\right)$ where $n \in {\Bbb N}$. Hence $i^i = \exp\left(\frac{-(4n + 1) \pi}{2}\right)$. – dr jimbob Jun 10 '14 at 12:57
  • 2
    @bof I thought "having nonzero imaginary part" was the definition of non-real. Some numbers are real, some numbers are not. Some numbers are imaginary, some numbers are not. The real numbers, imaginary numbers, and others, are all complex. Wikipedia agrees with me, for all that's worth...(not much) – Snowbody Jun 10 '14 at 13:06
  • 1
    pure imaginary numbers and real numbers form a direction sum decomposition of the complex numbers. For this to be natural, zero must be both real and pure imaginary. – James S. Cook Jun 11 '14 at 02:11
  • Is this really intuitively imaginary? Might one not intuit that it is complex with non-zero real part, and hence, not imaginary? Or might there be no intuition and thus, one only needs to make a simple calculation? – Mark Viola Mar 09 '15 at 19:18
  • @Dr.MV How can it be negative? – Akiva Weinberger Mar 28 '17 at 04:06
  • @AkivaWeinberger We have $$i^i\equiv e^{i \log(i)}=e^{i \times i(\pi/2+2k\pi)}=e^{-\pi/2-2k\pi} $$So, $i^i \in (0,\infty)$ – Mark Viola Mar 28 '17 at 04:09
  • ... and $i^i$ is rational – Frédéric Grosshans Nov 08 '19 at 07:02
62

This part is true (Jordan-Brouwer separation theorem):

(a) Any imbedding of the $2$-sphere into $3$-dimensional Euclidean space separates the space into two disjoint regions.

But this part, which would seem to be a natural generalization of the Jordan-Schönflies Curve Theorem, is not true:

(b) The regions are homeomorphic to the inside and outside of the unit sphere.

mjqxxxx
  • 37,360
  • 2
  • 50
  • 99
53

I really like "wrong proofs" as typically the insight why the proof is wrong gives you some understanding of the topic. One very simple version is this one, which I threw at my first semesters when I was a tutor:

Each binary relation which is symmetric and transitive is also reflexive and therefor an equivalence relation.

"Proof":

Let $\sim$ denote a symmetric and transitive relation and let $x$, $y$ be two elements with $x \sim y$. As $\sim$ is symmetric, it holds that $y \sim x$. Since $x \sim y$ and $y\sim x$ it follows by the transitivity of $\sim$ that $x \sim x$, which is the definition of reflexivity.

Edit: Since I was asked, here's why the proof is wrong (move your mouse there to show):

Take a look at the empty relation on a non-empty set $S$, so that there are no $x, y \in S$ so that $x \sim y$. This relation is symmetric and transitive, but it is not reflexive. Reflexivity needs $x \sim x$ to hold for all $x$. The proof assumes that there is a y so that x ~ y, which isn't necessarily the case for all $x$.

Michael Hardy
  • 1
  • 30
  • 276
  • 565
aRestless
  • 121
  • 1
  • 4
  • Where is the flaw in this argument? – beep-boop Jun 05 '14 at 22:33
  • @user21820 Sorry, for formulating too ambiguous. I edited and it should be clear now what's meant. – aRestless Jun 06 '14 at 10:39
  • Sorry I misread. – user21820 Jun 06 '14 at 10:58
  • 1
    A relation is not simply "reflexive"; rather it is reflexive on some set. The relation $\sim$ on $\{1,2,3,4\}$ where by $3\sim3$, $4\sim4$, $3\sim4$, and $4\sim3$, and nothing else is related, is not reflexive on the set $\{1,2,3,4\}$, but there is indeed a set on which it is reflexive. – Michael Hardy Jun 13 '14 at 18:18
  • This situation is dealt with in detail on page 30 of the book ESSENTIALS OF ABSTRACT ALGEBRA by Bundrick and Leeson (1972). – Mike Jones Jun 11 '15 at 16:32
  • 1
    The symmetric transitive relations comprise a very important class of relations, sometimes called the *partial equivalence relations*. Just as you can mod out by an equivalence relation to get a quotient, so you can restrict to and mod out by a partial equivalence relation to get a subquotient, that is a subset of a quotient (or equivalently a quotient of a subset). … – Toby Bartels Jan 12 '19 at 03:54
34

Theorems that are intuitively true, but actually flawed:

  • There is no continuous, nowhere-differentiable real function.

  • There is no real function that is differentiable and not monotonic on any non-trivial interval.

  • If a real function satisfies $\forall x, y, f(x+y) =f(x) +f(y) $, it is of the form $x\to ax$.

  • Infinite sums and integrals can be swapped anytime.

  • A connected metric space is path-connected.

Gabriel Romon
  • 32,348
  • 5
  • 56
  • 135
  • Why is intuitively true that infinite sums and integrals can be swapped anytime ? I think it's relatively simple to find a counter-example. –  Jun 04 '14 at 18:49
  • 9
    Can you show a counterexample for the third claim? (The one about linear functions) – Alessandro Codenotti Jun 04 '14 at 19:28
  • 1
    @alessandro see this http://math.stackexchange.com/questions/131197/graph-of-discontinuous-linear-function-is-dense/ – Gabriel Romon Jun 04 '14 at 19:30
  • 25
    Almost any statement where the counterexample begins with “Let $H$ be a Hamel basis of $\Bbb R$…” is going to be one of those seems-true-but-isn't things. – MJD Jun 04 '14 at 19:50
  • 2
    @Alessandro http://planetmath.org/thereexistadditivefunctionswhicharenotlinear and more detailed discussion at https://en.wikipedia.org/wiki/Cauchy%27s_functional_equation – MJD Jun 04 '14 at 20:10
  • I feel like the one about a linear function doesn't really fit. Isn't "linear function" **defined** by that property plus $f(ax) = a f(x)$? Wouldn't it only be non-obvious to someone who misinterprets the usage of "linear function"? – jpmc26 Jun 04 '14 at 23:38
  • @jpmc26 isn't the definition $f(ax+by)=af(x)+bf(y)$ or $f(ax)=af(x)$ and $f(x+y)=f(x)+f(y)$? – Alessandro Codenotti Jun 05 '14 at 09:01
  • "There is no real function that is differentiable and not monotonic on any non-trivial interval." Seriously? What is the counterexample? – goblin GONE Jun 05 '14 at 10:21
  • 1
    @user18921 http://www.jstor.org/stable/2318996?__redirected – Gabriel Romon Jun 05 '14 at 11:12
  • @Alessandro I said, "that property plus..." to refer to the one in the answer. – jpmc26 Jun 06 '14 at 04:33
  • 1
    For #4, the restriction where this _does work are given by the [Fubini-Tonelli theorem](http://en.wikipedia.org/wiki/Fubini's_theorem) – Mitch Jun 06 '14 at 17:18
34

Here's one of my favorites: Let's assume playing with a fair coin.

Theorem (false) In a long coin-tossing game each player will be on the winning side for about half the time, and the lead will pass not infrequently from one player to the other.

The following is from W. Fellers classic of Introduction to Probability Theory and It's Applications, Vol 1:

According to widespread beliefs a so-called law of averages should ensure the Theorem above. But, in fact this theorem is wrong and contrary to the usual belief the following holds:

With probability $\frac{1}{2}$ no equalization occurred in the second half of the game regardless of the length of the game. Furthermore, the probabilities near the end point are greatest.

In fact this leads to the Arc sine law for last visits (see e.g. Vol 1, ch.3, section 4, Theorem 1).

Note: Please note the remarkable statements cited from Chapter III: Fluctuations in Coin Tossing and Random Walks:

For example, in various applications it is assumed, that observations on an individual coin-tossing game during a long time interval will yield the same statistical characteristics as the observation of the results of a huge number of independent games at one given instant. This is not so.

and later on:

Anyhow, it stands to reason that if even the simple coin-tossing game leads to paradoxical results that contradict our intuition, the latter cannot serve as a reliable guide in more complicated situations.

[2015-07-16] According to a comment from @HenningMakholm some examples exposing striking aspects.

  • Suppose that a great many coin-tossing games are conducted simultaneously at the rate of one per second, day and night, for a whole year. On the average, in one out of ten games the last equalization will occur before $9$ days have passed, and the lead will not change during the following 356 days. In one out of twenty cases the last equalization takes place within $2\frac{1}{2}$ days, and in one out of a hundred cases it occurs within the first $2$ hours and $10$ minutes.

  • Suppose that in a learning experiment lasting one year a child was consistently lagging except, perhaps, during the initial week. Another child was consistently ahead except, perhaps, during the last week. Would the two children be judged equal? Yet, let a group of $11$ children be exposed to a similar learning experiment involving no intelligence but only chance. One among the $11$ would appear as leader for all but one week, another as laggard for all but one week.

The examples above are in fact a consequence of the Arc sine law for last visits.

epi163sqrt
  • 94,265
  • 6
  • 88
  • 219
  • Could you add some explanation to the third quote about how it can coexist with the usual shorthand assumption that a "coin toss" is independent of every other coin toss anywhere in spacetime? How would the coins observed in the long game "know" they are being used in a long game rather than carried out simultaneously in different games? – hmakholm left over Monica Jul 15 '15 at 20:49
  • @HenningMakholm: I will add some information. In fact behind the scene it reduces to a purely combinatorial aspect. There are more possibilities to preserve or increase the height within $k$ steps than to go down. Maybe we could informally say, that it's not the coin, which knows anything about its path, but instead the surrounding *space* with the specific lattice structure, which has a certain knowledge of the embedded coin and its current height. :-) – epi163sqrt Jul 15 '15 at 22:10
  • Probably the most needed information is what the "observations" they speak about are. As quoted it sounds like they claim you can hand them a list of 1000 coin tosses that either originates from one long game or from 1000 different games, and they'd be able to guess which with better than 1/2 chance, by some kind of statistical analysis. – hmakholm left over Monica Jul 15 '15 at 22:28
  • @HenningMakholm: I've added two examples which could be helpful. Best regards, – epi163sqrt Jul 16 '15 at 20:41
  • Note that "observations on an individual coin-tossing game during a long time interval will yield the same statistical characteristics as the observation of the results of a huge number of independent games at one given instant." may actually be (somewhat) true, if you do it the right way (i.e. by looking at exponential time scales). It comes from the path-convergence to a Brownian motion, scale invariance of the Brownian motion, and ergodicity/mixing of the rescaling flow of the Brownian motion. – D. Thomine Apr 08 '16 at 18:07
  • D.Thomine: Interesting information. Thanks! – epi163sqrt Apr 08 '16 at 20:03
  • Can you please provide a reference to the "_Chung & Feller Introduction to Probability Theory and It's Applications, Vol 1_" book that you suggest? I know [the classic book](https://books.google.co.in/books?id=A6J4AGUwrPwC&source=gbs_book_other_versions) by the same name but it is only by Feller and another one is _[Elementary Probability Theory](https://www.springer.com/gp/book/9780387955780#otherversion=9780387215488)_ by Chung and AitSahlia. – Apoorv Potnis Jan 20 '20 at 17:04
  • 1
    @ApoorvPotnis: I'm sorry, this was a mistake. I'm also referring to W. Fellers book. The arc sine law was first proved by K. L. Chung and W. Feller and it seems I mixed it up. I've updated the post. Best regards, – epi163sqrt Jan 21 '20 at 21:04
31

The following statement I once believed to be "obvious":

If $f:\mathbb{R} \rightarrow [0,\infty)$ continuous is such that $\int_{-\infty }^{\infty }f(x)\text{d}x<{\infty } $, then $\lim \limits_{x \to \pm\infty} f(x) = 0$

which is actually false.

(Note: It is true if $f$ is uniformly continuous!)

Luke Skywalker
  • 624
  • 1
  • 6
  • 19
  • 1
    Can you provide a counterexample? – dazedviper Jun 11 '14 at 22:26
  • 20
    This is pretty easy to give a counterexample to: let $f$ have bumps as far down as you like, just with sufficiently small widths so that the sum of their integrals goes to zero. You can make $\limsup_{x \to \infty} f(x) = \infty$ this way. – Dustan Levenstein Jun 12 '14 at 03:43
  • @hallaplay835 c.f. Stein&Shakarchi's Real Analysis book, the chapter on $L^1$ functions. It was an excercise. – Zhipu 'Wilson' Zhao Jun 12 '14 at 04:17
  • 3
    @hallaplay835 If $f(x) = \cos(x^2)$, then clearly $f$ doesn't even have a limit as $x$ approaches infinity. But the integral of $f$ does exist as an improper Riemann integral (Hint: Make a change of variable $y=x^2$ and $dx=\frac{1}{2\sqrt(y)}$ and Abel's test does the rest!). – Mark Viola Mar 09 '15 at 19:09
  • 1
    You can assume $f$ to be positive. That makes the condition a bit more misleading. In fact, this is exactly what is claimed in many quantum mechanics books in attempts to prove the decay of distribution function at infinity. – Hans Jun 07 '16 at 19:14
28

There are a good number of counterintuitive probability situations out there. One of my favorites is nontransitive dice:

There are 3 dice, A, B and C. The dice have numbers from 1-9 on their sides (repeats possible). If die B beats (higher number) die A more than half the time and die C beats die B more than half the time, then die C will beat die A more than half the time.

This is not necessarily a true statement. Dice can be designed such that the "x beats y" property is not transitive. A beats B, which beats C, which beats A.

Duncan
  • 720
  • 4
  • 6
  • 5
    There are a number of similarly counterintuitive statements in multiway voting, of which this could be seen as a special case. For example, in figure skating competitions at the championship level there are nine judges. It would sometimes happen that a majority of judges would rank skater $A$ above skater $B$, a majority would rank $B$ above $C$, and a majority would rank $C$ above $A$. The scoring rules had to have conditions in place to deal with this kind of situation. – MJD Jun 05 '14 at 21:35
  • In fact, you can come up with any crazy directed graph among the candidates that you want, and there is a population of apparently rational linear-order voters whose voting behavior will result in that outcome. It turns out that the ONLY behavior for voters that results in consistent majority decisions is blindly following a party line. – Nick Matteo Jun 07 '14 at 17:50
  • Is nine the minimum number of sides/judges requited that non-transitivity phenomenon can be achieved? – Akavall Jun 15 '14 at 02:20
  • Definitely not. Most non transitive dice are described as being 6-sided. It's just that the numbers on them are from 1-9 (not necessarily all used). – Duncan Jun 15 '14 at 05:28
  • @Duncan what's the minimum number of side for the dices? would you please quote an example of that voting? – athos Jul 07 '14 at 02:18
  • Minimum # of sides would be 3 (though it's tough to have a 3 sided die). But take these as examples anyway: die A: 2,4,9 die B: 1,6,8 die C: 3,5,7 Die A beats die B 5/9 of the time, die B beats die C 5/9 of the time, die C beats die A 5/9 of the time. (This is just the example from the wikipedia page on non-transitive dice but with the # of sides cut in half.) – Duncan Jul 07 '14 at 16:10
  • 2
    @duncan Suppose you have three skaters, X Y and Z. Judge 1 prefers X to Y and Y to Z; judge 2 prefers Y to Z and Z to X; judge 3 prefers Z to X and X to Y. Now two of three judges prefer X to Y; two prefer Y to Z, and two prefer Z to X. – MJD Mar 14 '15 at 17:33
26

The real numbers/Cantor set are countable.

There are several false "obvious" proofs:

  1. "Proof". Consider the tree $\{0,\ldots,9\}^\Bbb N$, then every real number corresponds to a node in the tree. Since there are only countably many levels and each is finite, it follows that the real numbers are finite.

    Why does it fail? This set is actually not a tree. You can order it so it looks like a tree, but in fact the tree would be composed of initial segments of each functions ordered by continuation. This tree, then, would have a last level (namely a level that no point there has a successor), and it would be exactly the level of the functions themselves (the previous levels would be proper initial segments of the functions).

    If we remove that last level, then the tree is indeed countable, but now each real number corresponds to a branch in the tree rather than a node. (It's the unique branch whose limit equals to the function, which previously appeared on that final level.)

  2. "Proof". The rational numbers are countable, and between every two real numbers there is a rational number. Therefore this defines a bijection between pairs of real numbers and the rational numbers.

    Why does it fail? Because there are many, many, many pairs being mapped to the same rational number, this is not actually a bijection.

  3. "Proof". The Cantor set is closed, its complement is open, so it is a countable union of intervals, so the Cantor set is countable.

    Why does it fail? Because not every point in the Cantor set is an endpoint of such interval. For example $\frac14$. It is true that the endpoints of these intervals form a countable dense subset, though.

  4. BONUS!, $\mathcal P(\Bbb N)$ is countable.

    "Proof". For every finite $n$, $\mathcal P(n)$ is finite, and $\mathcal P(\Bbb N)=\mathcal P(\bigcup n)=\bigcup\mathcal P(n)$, is a countable union of finite sets, which is countable.

    Why does it fail? Because the union only includes finite subsets of $\Bbb N$, but none of its infinite subsets.

Asaf Karagila
  • 370,314
  • 41
  • 552
  • 949
  • 4
    I think the non-existence of irrational numbers may itself be in this category (obviously true, but false), but was discovered to be false so long ago that we no longer remember how bizarre it must have seemed. – MJD Jun 04 '14 at 19:29
  • @MJD: True, and in two centuries, they will find it strange that at some point people thought that $\Bbb R$ can be countable. Or at least that what I hope! – Asaf Karagila Jun 04 '14 at 19:33
  • Now it strikes me as odd that nobody before Cantor observed that if $\Bbb R$ were countable, it could be covered by a family of disjoint intervals whose total length was less than any given $\epsilon$. – MJD Jun 04 '14 at 19:39
  • @MJD: Hmm, it is a bit odd. Maybe they executed all those people who suggested that before they could publish their finding! ;-) – Asaf Karagila Jun 04 '14 at 19:44
  • In which sense does your tree have a “final level” and what does that term even mean here? – Christopher Creutzig Jun 04 '14 at 21:51
  • @Christopher: It's not even a tree, strictly speaking. But when presenting this proof one often argues that the tree is the initial segments of a function from $\Bbb N$ to $\{0,\ldots,9\}$. The final level is the function itself. And it means that these nodes have no successors (and they are not successors themselves of any other node). – Asaf Karagila Jun 04 '14 at 21:54
  • Since the decimal expansion of, say, $1/3$, does not have a last digit, the “tree” does not have a final level in this sense. – Christopher Creutzig Jun 05 '14 at 04:48
  • 2
    @Christopher: No, the tree has $\omega+1$ levels. The last is the infinite sequence $0.333\ldots$. – Asaf Karagila Jun 05 '14 at 06:27
  • I have made mental peace with the fact that $\mathscr P(\mathbb N)$ is uncountable long ago. In fact, it has become so ingrained in my mind that I find hard to accept that the set of _finite_ subsets of $\mathbb N$ _is_ countable! – triple_sec Jun 05 '14 at 11:04
  • @triple_sec: Haha, yeah, that can happen too. Although it's one of these things that if you think about it for a little while, they become apparent again. – Asaf Karagila Jun 05 '14 at 11:06
  • @Asaf Hilariously, with your $\omega + 1$ you have used the idea of the common misconception "$0.999\dots < 1$ because they differ by an amount $0.000\dots 1$" in a correct argument about another, closely related, famously doubted theorem. – Ryan Reich Jun 05 '14 at 15:22
  • @Ryan: I don't see how. The objects in the tree are not real numbers. They are functions from $\Bbb N$ to $\{0,\ldots,9\}$. We can identify them as real numbers, but this identification need not be one-to-one; just onto. And that's exactly what's going on here. Also, the elements on the $\omega+1$ is not a sequence of order-type $\omega+1$, but rather a sequence of order type $\omega$. – Asaf Karagila Jun 05 '14 at 15:27
  • @Asaf: I didn't mean that you actually used decimal sequences of type $\omega + 1$, but simply that you constructed real numbers, as decimals, via an inductive process with $\omega + 1$ steps, where each of the first $\omega$ is a finite decimal to which the mistaken belief applies correctly. – Ryan Reich Jun 05 '14 at 15:31
  • @Ryan: Again, that's not what I did. I didn't construct *any* real number. I merely said that there is a way to identify every real number as a node in the tree. Some may have more than one way. – Asaf Karagila Jun 05 '14 at 15:34
  • @Asaf, I fail to see how you get order type $\omega+1$ entries in elements of the set $\{0,\dots,9\}^\mathbb{N}$. E.g., I think we agree there is exactly one element in this set corresponding to the decimal expansion of $\pi-3$. If that element, as you say, has order type $\omega+1$, then it has a last digit. Which one is that and why? And how does it come to be in a function from $\mathbb{N}$ to $\{0,\dots,9\}$? – Christopher Creutzig Jun 05 '14 at 20:23
  • @Christopher: The height of the tree is $\omega+1$. Not $\omega$. To be explicit, let me define exactly what is the tree, the $n$-th level (where $n – Asaf Karagila Jun 05 '14 at 20:30
  • @Asaf, maybe you could clarify what exactly you mean by “height” here. Each element of the set you defined clearly has order $\omega$, and while I believe we mean the same thing for “the $n$-the level” when $n$ is finite, if I read $f\upharpoonright n$ as $f$ limited to the preimage set $\{0,\dots,n\}$. Now, none of these actually is in your set. Each element of your set is the union of a countable number of such finite sequences, which you can choose to view as finite paths. But what would, with your definition of “height,” be such a tree of height $\omega$? – Christopher Creutzig Jun 05 '14 at 20:59
  • @Christopher: Yes, that is exactly what is written in my answer, if you care to read it. The set $\{0,\ldots,9\}^\Bbb N$ is **NOT** a tree, but we can think of it as a set which behaves a bit like a tree in some sense. But the actual tree is the restriction of each function to an initial segment of the domain. There are $\omega$ *proper* initial segments, and then there is one more initial segment which is the actual function. This defines a tree which has height $\omega+1$. And by the height of the tree I mean the order type of its levels, since there are $\omega+1$ levels, that's the height. – Asaf Karagila Jun 05 '14 at 21:11
  • @MJD Wasn't the observation that $\mathbb{R}$ is not countable necessary for Cantor's observation? – Brilliand Jun 06 '14 at 00:44
  • @Brilliand: That *was* Cantor's observation. – Asaf Karagila Jun 06 '14 at 13:07
  • All this confusion about non-proof 1. Can't we just ask which natural number is supposed to correspond to $\pi$? – Keen Sep 30 '14 at 19:26
  • 1
    @Cory: Or $\frac13$, yes. There are many examples. But this doesn't quite explain why the proof fails. It just shows that it does. I was trying to actually explain the reason for the failure. – Asaf Karagila Sep 30 '14 at 19:32
26

Hypothesis: Every infinitely-differentiable function is real-analytic somewhere.

This is false, as shown by (for example) the Fabius function.

Jesse Madnick
  • 29,652
  • 7
  • 90
  • 147
26

I'm surprised noone gave this answer already, so here it is:

There are more integers than there are natural numbers.

It's obvious, isn't it?

GOTO 0
  • 1,452
  • 1
  • 16
  • 23
  • 16
    I hesitate with this one, because it depends on (somewhat unsound) definition of "more". – DanielV Jun 07 '14 at 16:24
  • 13
    There absolutely are more integers than natural numbers (by inclusion). And cardinality is a slightly contrived notion if you think about it for a moment. –  Jun 08 '14 at 09:51
  • 25
    @Tibor: No, cardinality is *not* a contrived notion if you think about it for a *long* moment. – Asaf Karagila Jun 08 '14 at 21:09
  • 15
    All of mathematics is contrived, by definition. :) – Ryan Jun 09 '14 at 01:32
  • 1
    “I'm surprised no one gave this answer already”. Actually, the fourth subparagraph of [this answer](http://math.stackexchange.com/a/821753/121411), “There are just as many even numbers as natural numbers.”, was posted Jun 5 at 13:58 (implicitly, it is listing things that are true but non-obvious), and [this comment](http://math.stackexchange.com/questions/820686/obvious-theorems-that-are-actually-false/822137?noredirect=1#comment1700465_822137), “… there are half as many odd integers” [as integers], was posted Jun 6 at 7:30. – Scott Jun 09 '14 at 21:40
  • These are essentially equivalent to your post: the claim that, if a one-to-one correspondence exists between one (non-empty) set and a strict subset (in particular “half of”) another (non-empty) set, then there cannot be a one-to-one correspondence between the sets in their entirety. As I mentioned in another comment, I believe that this is true if the sets are finite, which is what makes it obvious. – Scott Jun 09 '14 at 21:41
  • I agree with GOTO o because not only the fact the integers are basicaly a better name for whole numbers and natural numbers are only the postive whole/integers but also because you can caulcate how many integers they are compared to natural numbers if we let i be the number of integers and n be the number of natural numbers and we get... i = 2n + 1 and n = (i - 1)/2... because there as many natural numbers as negative intgers (which is basicaly the opposite and there must be the same number of opposites as regulars) + 0 which is two n's plus 0 which counts as an extra number. – Binary Freak Jul 23 '14 at 19:57
  • First it was strange that there are just as many natural numbers as integers. But over the centuries, people got used to that, until it came as a shock that there really are more real numbers than rational numbers! – Toby Bartels Jan 12 '19 at 04:46
22

Image of a measure zero set under a continuous map has measure zero!

adrido
  • 2,133
  • 16
  • 25
  • Isn't is true?!?! – MR_BD Feb 12 '17 at 07:07
  • 2
    @LeilaHatami No, see [here](https://math.stackexchange.com/questions/480172/must-the-continuous-image-of-a-null-set-be-null). Essentially, we can bijectively (and continuously) map the cantor set to [0,1], which maps a measure 0 set to a measure 1 set. – Mark Feb 22 '17 at 07:30
19

The probability that you hit any single point on a dart board is $0$ but the probability that you hit the dart board is $1$ (as long as you're not as bad as I am at throwing darts ;D).

EDIT:

As @JpM pointed out I didn't follow the format of these posts albeit the idea can (easily in my opinion) be understood from what I've said above.

Pseudo-Claim: The probability of hitting a single point on a dart board is greater than $0$ since the probability of hitting it at all (assuming that you will hit the dart board) is $1$.

Seems obvious in the sense that a bunch of $0$ can't add up to be $1$ so each point must have some probability. Actually false because of some properties of measures.

DanZimm
  • 5,481
  • 1
  • 19
  • 36
  • ...is this not an unobvious theorem that is actually true?? – JP McCarthy Jun 05 '14 at 07:41
  • @JpMcCarthy lay-people might claim "the probability we hit a single point is greater than $0$" then indeed this is a theorem that is actually false. – DanZimm Jun 05 '14 at 07:43
  • 7
    In particular, there's an "obvious" intuition that the point *you actually did hit* must have had probability greater than $0$, "since probability $0$ events are impossible". I guess that this example is *so* obvious that it has to be made axiomatically false in the theory in order to get it out of the way and do some work :-) – Steve Jessop Jun 05 '14 at 08:27
  • What if I claim that the probability of hitting it is a non-archimedian hyperreal number? – DanielV Jun 05 '14 at 12:14
  • 1
    @DanielV I would say one of two things based on my mood: a) A lay-person would not claim such a thing, i.e. they wouldn't say the probability of hitting a single point is an infinitesimal. b) Generally (in fact I have not seen otherwise, then again I'm young and naive) measures are defined with values in the extended real numbers, meaning without infinitesimals. – DanZimm Jun 05 '14 at 12:23
  • I am confused. Maybe you mean to say "The probability that you hit any single point on a dart board is non-zero"?? That would be obvious and false. – JP McCarthy Jun 05 '14 at 12:50
  • @JpMcCarthy that's exactly what I said in the comment. – DanZimm Jun 05 '14 at 12:52
  • ...then that should be your answer! Your answer is an unobvious theorem that is actually true rather than an obvious theorem that is actually false. – JP McCarthy Jun 05 '14 at 12:58
  • 1
    @JpMcCarthy ok I changed the format of my post to follow exactly how the question was asked. – DanZimm Jun 05 '14 at 13:03
  • @JpMcCarthy Thanks ;D I like the ones above mine more, but this was an interesting one. – DanZimm Jun 05 '14 at 13:07
  • Should it be assumed that "point" means "region on the dart board such that the area is 0" (or perhaps "0-dimensional region on the dart board", though the property would hold true for a 1-dimensional line as well), which would not necessarily coincide with the layman's definition of "point"? – JAB Jun 09 '14 at 19:03
  • @JAB I think you're overcomplicating this, a layperson is not going to think this in depth about the definition of "point". I'm considering a point to be exactly what a layperson would think, one exact place on the dart board. – DanZimm Jun 10 '14 at 00:31
  • 3
    I think the essential thing some people simply can't get their head around is that an event with probability 0 _can still happen._ – Neil W Jun 11 '14 at 02:43
  • @Neil yep that's exactly what I was trying to get at here ;D – DanZimm Jun 11 '14 at 02:48
  • This is not a mathematical statement, nor is it false. – jwg Jun 11 '14 at 11:56
  • Isn't this analoguous to that an integral from a to a of an integrable f for 0 < a < 1 must be > 0 if the integral of f from 0 to 1 is non-zero? – scibuff May 07 '15 at 07:04
  • @scibuff in fact it's equivalent, given the correct setting – DanZimm May 07 '15 at 07:24
19

My "theorem":

The statement Everybody loves my baby, but my baby loves nobody but me is about a pair of lovers

It is so simple and so obvious, even my grandma will understand it. And no matter how much you explain the simple logic calculation which shows that we are talking about a single narcissist here, half the class of first-semester logic students will continue insisting that your proof is wrong, and they don't know what is wrong about it, but it cannot refer to a single person.

rumtscho
  • 756
  • 5
  • 20
  • 10
    Some people treat love as a relationship that is not defined reflexively. – Joshua Jun 14 '14 at 02:16
  • 1
    @Joshua I have to yet meet a student who notices that this is the root of the problem, and explain how to avoid it in the logical "proof". You are right, of course, once we define the set of "everybody" not as "all humans" but "all humans who are capable of loving my baby", and decide that my baby is not in the set, the result is different. – rumtscho Jun 15 '14 at 17:19
  • 17
    I have no idea what this answer is trying to say. – goblin GONE Sep 17 '15 at 00:45
  • 6
    @goblin As can be seen from this [proof in logic](http://members.ozemail.com.au/~markhurd/NobodyButMe.html) the only possible meaning of the statement is that __my baby is me__. – Mark Hurd Jun 04 '16 at 13:44
18

Consider a function $f:(0, \infty) \rightarrow \mathbb{R}$ that is $\mathcal{C}^\infty$ on that interval. At first glance, one might think that, if $\lim(f) = 0$ as $x \rightarrow \infty$, then $\lim(f') = 0$ as $x \rightarrow \infty$. However, this is false. Here is but one counterexample:

$$f(x) = \frac{1}{x}\sin(x^2)$$

Further, if we add the stipulation that $f$ also be monotonic, counterexamples can still be found (though they are quite pathological).

Kaj Hansen
  • 31,861
  • 4
  • 53
  • 94
  • 1
    Your "another" was already mentioned......... –  Jun 04 '14 at 19:34
  • Oops, didn't see it there. Thanks. – Kaj Hansen Jun 04 '14 at 21:15
  • 2
    Caould you give a $ \mathcal{C}^\infty$ monotonic, counterexamples? Thank you very much! – 2016 Jun 05 '14 at 13:49
  • 2
    Yes, you can get one by tinkering with the responses here a bit: http://math.stackexchange.com/questions/788813/the-limit-of-the-derivative-of-an-increasing-and-bounded-function-is-always-0/788842 – Kaj Hansen Jun 06 '14 at 02:35
17

A simple arc (homeomorphic image of the closed unit interval) in the plane has $2$-dimensional Lebesgue measure zero.

bof
  • 70,356
  • 5
  • 80
  • 153
  • this is obvious right -- isn't it? – athos Jul 07 '14 at 02:32
  • 3
    @athos It **should** be obvious, otherwise it wouldn't be a valid answer to this question. Are you asking why it's actually false? There is probably a simpler direct construction of a "fat arc" in the plane, but it follows from a more general theorem of R. L. Moore and J. R. Kline ([On the most general plane closed point-set through which it is possible to pass a simple continuous arc](http://www.jstor.org/stable/1967872), Ann. of Math. (2) 20 (1919), 218-223) that every (homeomorph of the standard) Cantor set in the plane is contained in an arc; apply this to a fat Cantor set in the plane. – bof Jul 07 '14 at 03:28
  • thanks for the link... a bit tough for me, will bookmark it for later digestion :) – athos Jul 07 '14 at 03:52
17

Any real number can be computed somehow.

More formally:

For every real number, there exists a finite-length program that computes that number.

Since real numbers are uncountable while computable numbers are countable, that just can't be the case.

This limitation comes from the fact that we're stuck using finite-length programs. Infinite-length programs can be defined to compute any real number (trivially). So there is a sense in which all real numbers can be computed.

Just not by humans. Note that, since a single infinite-length program would take up infinite memory (and we don't seem to have any infinite computers/brains), the majority of these infinite-length programs can never be known, let alone computed. So computable numbers are only those numbers computable by a finite-length program. And the set of finite-length programs is countable.

Keen
  • 192
  • 8
  • What I personally think it should be is... For every nontransdental, real number there exists a finite-length program that computes that number... – Binary Freak Jul 25 '14 at 03:21
  • @BinaryFreak I believe the non-transcendentals are all algebraic and thus computable, so that would be an obvious theorem that turns out to be true. On an interesting related note, while every non-computable number is transcendental, some transcendentals are computable ($\pi$, $e$, and some non-repeating reals like $0.123456789101112...$). – Keen Jul 25 '14 at 15:43
16

Theorem: Let $f_1(x,y)$ and $f_2(x,y)$ be two joint probability densities, each having its $x,y$ components positively correlated ($Cov_1(x,y)>0$, $Cov_2(x,y)>0$). Let $f_3=\alpha f_1 + (1-\alpha) f_2$ be the mixing density, for some $0\le \alpha\le 1$. Then $Cov_3(x,y)>0$.

In words: mixing populations preserves the correlation sign. In other words: if the average MSE male user is brighter than the mean, and if the average MSE female user is brighter than the mean, then the average MSE user is brigther than the mean. Obviously true.

False. See Simpson's paradox.

leonbloy
  • 56,395
  • 9
  • 64
  • 139
  • 1
    Perhaps the following example will make the obviousness more obvious. Baseball team $A$ has a better win-lose ratio than team $B$ in the first part of the season. Then there is a strike, and some games are missed. When the season resumes, team $A$ also has a better win-lose ratio than team $B$ in the second portion of the season. Therefore, team $A$ has a better win-lose ratio than team $B$ overall. (Wrong!) Baseball did play such a split season in 1981, but I don't know if the paradox actually occurred then. Probably not, as it requires $A$ and $B$ to play very different numbers of games. – MJD Jun 06 '14 at 00:43
15

Claim:

If the dot product of two vectors is 0, then they are linearly independent.

My prof threw this question at me today and I fell for it.

  • 2
    haha, took me a second. – Bennett Gardiner Jun 11 '14 at 02:47
  • depends on what field. very nice, but a trick question. I feel cheated. – Lost1 Jun 12 '14 at 11:29
  • 6
    @Lost1: It actually doesn't depend on the field: $\vec{v}\cdot \vec{0} = 0$ and $\{\vec{v}, \vec{0}\}$ is never independent. – Jason DeVito Jun 12 '14 at 15:52
  • 2
    @JasonDeVito ah... i was thinking take $\mathbb{Z}_2^2$ and the vector $(1,1)$... – Lost1 Jun 12 '14 at 17:46
  • @Lost1 According to the usual definition, the dot product must be real-valued (certainly not finite-field valued; they are not ordered fields, and without order, positive-definiteness makes no sense. And without that, this theorem is false but also has no reason to be true!) – Ryan Reich Jun 16 '14 at 04:25
  • @RyanReich Not necessarily. It is quite standard in linear algebra (and number theory) to consider inner products taking values in the underlying field of the vector spaces under consideration. Only when specializing to real or complex valued fields does positive definiteness enter the picture. Since now we may have $v\cdot v=0$ for $v\ne0$, the notion of [*isotropic*](http://www.encyclopediaofmath.org/index.php/Isotropic_vector) vectors is used. – Andrés E. Caicedo Jul 02 '14 at 17:50
14

In my opinion, the most interesting (but also sometimes not intuitive) results in mathematics are those that state a theorem that ends up being false because it actually holds in many cases, except for very few or very strange cases. In other words, the most "obvious" false theorems to me are those that have very difficult counterexamples.

Some examples:

  • Banach-Tarski: There exists a strict subset $A$ of the Euclidean $n$-ball $B$ such that one can partition $A$ and $B$ into an equal number of further subsets that can be mapped to each other by isometries. This shows that not all sets are measurable, and that it's possible to perform partitions that do not preserve measure.

  • Non-finiteness of differentiable structures: For $\mathbb{R}^n$ with $n = 4$, there are an uncountable number of distinct differentiable structures.

  • Divergence of Fourier series: There exists an integrable function on $[-\pi, \pi]$ whose Fourier series diverges everywhere. This is extremely unusual because for any typical function we might write down, usually its Fourier series might diverge at one or a finite number of points, but will probably converge everywhere else.

Christopher A. Wong
  • 21,201
  • 3
  • 44
  • 79
  • This makes me think of: a subset of $\mathbb{R}^n$ is either closed or open (but not both). Which of course is not true because the empty set is open and closed. (But also, I don't think $\mathbb{Q}$ is either closed or open in $\mathbb{R}$). – Ruben Jun 04 '14 at 19:36
  • 12
    You don't need a weird example to find a subset of $\Bbb R^n$ that is neither open nor closed; just take a closed ball, and remove a boundary point; or take a sequence of points that converges to a limit, but omit the limit. The *terminology* may lead one to think that sets are either open or closed, but there is nothing about the actual concepts that would make it appear so. – MJD Jun 04 '14 at 20:07
  • What is an example of function with everywhere divergent Fourier series? – Ruslan Jun 05 '14 at 11:59
  • @Ruslan, The example is nontrivial and is due to Kolmogorov. An earlier, almost-everywhere divergent construction was one of Kolmogorov's first claims to fame. You'll have to consult Google on this one, I do not know the example. – Christopher A. Wong Jun 05 '14 at 19:14
  • @Ruslan Google leads us back [here](http://math.stackexchange.com/q/14855/820). – Mark Hurd Jun 06 '14 at 09:44
14

An 'obvious' but false theorem: There are more open sets in $\mathbb R^2$ (or $\mathbb R^n$) than there are real numbers.

And in a similar vein we have this corollary to the first statement: There are more continuous functions $\mathbb R\rightarrow\mathbb R$ than there are real numbers.

(Both statements are false.)

paw88789
  • 36,857
  • 2
  • 28
  • 64
13

Here are some of the false statements popping into my mind that made me raise at least one eyebrow when I first realized they were not true.


Every linear function between two vector spaces is continuous.

True only as long as the domain is finite-dimensional. If it is not, then there exists a linear function that is not continuous—at any point!


The set of real numbers can in no way be (totally) ordered in such a way that every non-empty set in it has a least element.

False if choice is assumed, by the well-ordering theorem.


$\mathbb Q$ is not countable.

I am still tempted to believe it sometimes...


If the derivative of a continuous real-to-real function exists almost everywhere and (wherever it exists) vanishes almost everywhere, then the function must be constant.

False. In fact, there exists a function that satisfies the premise and it is strictly [sic!] increasing!


Any compact set is closed.

The name “compact” would suggest this, but this can be guaranteed only in Hausdorff spaces.


A set is compact if and only if every sequence in it contains a convergent subsequence.

While true in metric spaces, not only is it false in some more general topological spaces, but also neither condition implies the other!

triple_sec
  • 22,017
  • 3
  • 33
  • 79
  • 4
    I totally believe that $\mathbb{Q}$ is countable. Did you mean $\mathbb{R}$? For the next, do you assume that the derivative exists everywhere, or only almost everywhere? – Daniel Fischer Jun 05 '14 at 09:14
  • No, I meant $\mathbb Q$. I mean, duh, $\mathbb Q$ _is_ countable, but I always need a leap of faith to believe it. If I can enumerate the rationals in $[0,1]$, why can't I do that in an increasing order? (That is, such that $q_1 – triple_sec Jun 05 '14 at 10:10
  • 2
    I've convinced myself that $\mathbb Z^2$ is countable; obviously it is possible to devise an algorithm to count them. And I've convinced myself that this means that $\mathbb Q$ is countable, because $\mathbb Q$ is ordered pairs of elements in $\mathbb Z^2$, so there are "less" elements in $\mathbb Q$ than $\mathbb Z^2$ (thinking of graphing elements, $\mathbb Q$ leaves holes because $\frac 24 = \frac 12$), so $\mathbb Q$ is definitely not uncountably infinite. – Justin Jun 06 '14 at 02:51
  • @Quincunx You're perfectly right. It just rings _intuitively_ very odd to me. For example, say that $f:\mathbb R\to\mathbb R$ is a function. Suppose tha $f$ is continuous except at a countable set of points. You may be tempted to think that this function is “almost well-behaved,” right? It turns out that there exists a function that is discontinuous precisely at all rational points. At this point, I would conclude that the function is not “almost well-behaved” at all, but rather pathological. After all, the set of discontinuities of $f$ is dense! – triple_sec Jun 06 '14 at 03:12
  • 1
    I guess the one feature of $\mathbb Q$ that throws me off balance is that it just somehow doesn't feel “right” to me for a countable set to be dense in an uncountable one. – triple_sec Jun 06 '14 at 03:14
12

The Hauptvermutung states that there is essentially only one PL structure on a manifold. More precisely, it states that any two triangulations have a common subdivision. The reason why this seems "obviously true" is that you can take both triangulations and superimpose them one on top of the other, subdividing the manifold into a bunch of cells, and then taking the barycentric subdivision to get a triangulation. It turns out that this is false and one needs some pretty subtle invariants to detect it. The problem with the argument that I gave is that one triangulation could be very wild with respect to the other (fractally wiggly) so that their union does not subdivide the manifold into a nice collection of cells.

Cheerful Parsnip
  • 25,771
  • 2
  • 51
  • 117
  • For what sorts of manifolds does it fail? – MJD Jun 05 '14 at 20:39
  • @MJD As I recall, there is an invariant of the fundamental group (called the Whitehead group) which is defined in terms of matrices over the group ring, and this needs to be nonzero for the manifold to stand a chance of being a counterexample. This rules out a lot of fundamental groups, including the trivial group and $\mathbb Z$. It's been a while since I've thought about this. – Cheerful Parsnip Jun 05 '14 at 20:46
11

Stein's paradox is to me the most puzzling mathematical notion I've ever known (although I'm not a mathematician), mostly because it's not a mathematical "artifact", but its non-intuitiveness carries very tangible error consequences.

Theorem: (false)

One can do no better than ordinary decision rule for estimating the mean of a multivariate Gaussian distribution under mean squared error.

In other words, completely independent phenomena can actually be combined for a lower joint estimation error.

10

What about this:

$\mathbb{R}$ and $\mathbb{R}^2$ are not isomorphic (as Abelian groups with addition).

It falls under the category of "Let's take the Hamel basis of $\mathbb{R}$...", but I like it a lot.

mbork
  • 339
  • 2
  • 8
9

I think this is not covered in any of the other answers (although, to be sure, there are a lot of them). The Simpson's paradox one is close, but I think this is different and somewhat easier to understand:

If $X$ is positively correlated with $Y$, and $Y$ is positively correlated with $Z$, then $X$ is positively correlated with $Z$.

In other words, positive correlation is transitive. I think it's fairly intuitive, yet false.

Brian Tung
  • 31,234
  • 3
  • 33
  • 64
  • It is true, however, that if both $X$ and $Z$ are positively and *perfectly* correlated with $Y$, then $X$ is positively and perfectly correlated with $Z$. – user1551 Aug 10 '17 at 17:00
9

"Obviously" $$(x^y)^z = x^{y\cdot z}$$ for $x,y,z \in \mathbb{C}$ such that given expressions are defined.

Sil
  • 13,256
  • 3
  • 31
  • 70
  • Someone needs to tell this to the authors of college-level remedial elementary-algebra textbooks. These consistently state that as a theorem when x is real and y and z are rational. – Toby Bartels Jan 12 '19 at 11:29
9

"A sequence of numbers in which every number is larger than the previous, will always eventually go above a given value L."

CaptainCodeman
  • 1,587
  • 9
  • 11
9

I really want the following to be true:

Theorem: Let $S$ a subset of a vector space. If $S$ is pairwise linearly independent (meaning each $\{v,w \} \subseteq S$ is linearly independent) then $S$ is linearly independent.

And yet, it is false. For example, $$ \{ v,w,v+w \} $$ If $S$ only had two elements then we win by default. In any event, students tend to believe this. I mean, it's linear algebra, the principle of superposition ought to apply right? Something is the sum of its parts, linear independence begets linear independence... very seductive, very wrong.

James S. Cook
  • 16,540
  • 3
  • 42
  • 100
  • Is there such thing as a vector space where all pairs of vectors are linearly independent? Maybe you mean a vector space basis? Or is conflating vector space and a basis for a vector space common practice ? – DanielV Jun 11 '14 at 07:49
  • @DanielV, not my meaning, I will reword, I meant that if every pair of vectors in $S$ is linearly independent... thanks for the comment. – James S. Cook Jun 11 '14 at 11:33
  • I like your example, I just wanted to make sure I wasn't misinterpreting it. If I recall correctly, I see "vector space" to refer to the span of basis vectors, so two different basis vector sets may have the same vector space. For clarity I probably would have just stated "If every pair of vectors in S is linearly independent, then S is a set of linearly independent vectors". Sorry to criticize though, I believe that even a graphical description of this claim might take some students by surprise so I think it's a good example. – DanielV Jun 11 '14 at 12:50
8

Something I used to be seduced by in my mathematical immaturity (which is sadly still existing):

Suppose that $P_n$ are a family of statements indexed by $n\in\mathbb{N}$ and we can assign meaning to $P_{\infty}$. Then if $P_n$ is true for all $n\in\mathbb{N}$, then $P_{\infty}$ is true also.

JP McCarthy
  • 8,008
  • 1
  • 31
  • 55
  • That all depends upon how accurately you define the "meaning" of $P_{\infty}$ . – Mark Hurd Jun 06 '14 at 14:27
  • "Suppose that... we can assign meaning to $P_{\infty}$." I am assuming that we can. – JP McCarthy Jun 06 '14 at 14:46
  • In that case, I call for an example where the $P_\infty$ is defined "correctly" but is actually false. – Mark Hurd Jun 06 '14 at 16:01
  • 1
    The example I had in mind when I wrote this was $P_n$: sum of $n$ differentiable functions is differentiable; $P_\infty$: infinite sum of differentiable functions is differentiable. – JP McCarthy Jun 06 '14 at 16:09
  • 5
    How about the statement that 1/n > 0 which holds for any integer n but fails in infinity? –  Jun 06 '14 at 16:37
  • 1
    @JónÁskellÞorbjarnarson Mark would claim that $1/\infty>0$ is not meaningful because $1/\infty$ is not defined. An easier example is just a sequence of rationals converging to an irrational. – JP McCarthy Jun 06 '14 at 16:40
  • @Jp: We can clearly *assign meaning* to $1/\infty$. It's even useful. It's only "not defined" in the same sense that an ordinary sum of infinitely many things is not defined, or that for a family $P_n$ with $n \in \mathbf{N}$, $P_\infty$ is not defined. –  Jun 08 '14 at 21:15
  • 1
    @Hurkyl I agree and was trying to argue from Mark's position. – JP McCarthy Jun 08 '14 at 22:17
  • 1
    This is Leibniz's Law of Continuity, right? – Tanner Swett Jun 09 '14 at 04:29
  • 1
    @TannerSwett Following a quick Google, I do believe it is. – JP McCarthy Jun 09 '14 at 06:08
8

One of the first times I got caught out being wrong about something so obvious was believing:

abs(x) is never equal to -x

Of course abs(x) is defined as -x for x < 0

Old Pro
  • 354
  • 2
  • 14
  • 2
    A related non-theorem that my students want to believe is that $\sqrt{a^2}=a$. (Then again, I have students who want to believe that $\sqrt{a+b}=\sqrt a+\sqrt b$.) – Toby Bartels Jan 12 '19 at 05:09
6

Cauchy's theorem implies that:

if one makes a physical model of a convex polyhedron by connecting together rigid plates for each of the polyhedron faces with flexible hinges along the polyhedron edges, then this ensemble of plates and hinges will necessarily form a rigid structure.

However, there are counter-examples if you allow a general polyhedron (not convex).

Peter de Rivaz
  • 287
  • 2
  • 13
5

Before 1955 everyone “knew” that to know the nth decimal digit of $\pi$ (and for any other irrational) it was necessary to know the previous digits. A genius like Archimedes ("There was more imagination in the head of Archimedes than in that of Homer": Voltaire) "knew" very well this as History shows. However, The Bailey–Borwein–Plouffe formula (BBP formula) finished with this sacred for centuries “knowledge” and now it is possible to know, for example, the 33-th digit without to know the precedent ones.

Concerning the intuitive perception, it is false that a continuous numerical function must be derivable at least in one point; it is false too that a little square cannot contains a curve of infinite length.

Piquito
  • 25,368
  • 3
  • 24
  • 47
  • 2
    The BPP formula for pi doesn't give decimal digits, but hexadecimal digits. – Gerry Myerson Dec 05 '15 at 11:30
  • I think "1955" here must be an error. – MJD Nov 08 '19 at 10:14
  • The idea that "a little square cannot contain a curve of infinite length" becomes less "obvious" when people realize that each tiny cell in their bodies contains very long strands of DNA. (Of course the DNA isn't quite infinitely long and the cell is 3-dimensional, so this isn't a disproof of the "obvious" idea, but it seems to ruin the obviousness.) – Andreas Blass Sep 05 '20 at 15:00
5

EDIT: The counterexample I had in mind is incorrect; I've asked another question to try and clarify the matter one way or the other. Regardless, I think it's safe to say that this is no longer obvious! But I'll go ahead and update (or delete) my answer accordingly once I've gotten a bit more clarity.

Here's a topological example that takes some thought to falsify: roughly, 'for every non-intersecting curve between two opposite corners of a square, there's a curve between the other two corners that only intersects it once'. Formally:

Let $f: [0, 1]\mapsto [0,1]^2$ be a non-self-intersecting curve with $f(0) = (0,0)$, $f(1) = (1,1)$, and $f(t)\in (0,1)^2$ for $t\in(0,1)$. Then there exists a non-self-intersecting curve $g: [0, 1]\mapsto [0,1]^2$ with $g(0) = (1,0)$, $g(1) = (0,1)$, and $g(t)\in (0,1)^2$ for $t\in(0,1)$ such that there are unique $t_0$ and $ t_1$ with $f(t_0) = g(t_1)$.

This seems obvious (at least to me) on first glance, and even on second glance, the example of the Jordan Curve theorem suggests that it should be true; after all, we get a 'left side' and a 'right side' of our curve by the JCT, and doesn't the Schoenflies theorem mean that we should be able to find an inverse mapping of our curve to the circle? But it's false; there are curves $f()$ that can't be intersected only once by any curve $g()$. Finding a counterexample makes a nice exercise...

Steven Stadnicki
  • 50,003
  • 9
  • 78
  • 143
  • Are you sure? I think, I can prove that such g always exists. – Moishe Kohan Jun 04 '14 at 19:53
  • @studiosus I'd love to see your proof! I was fairly confident in my example, but on further thought I'm actually willing to believe that it can be falsified. I may not have been considering sufficiently degenerate $g()$ to go with my canonically degenerate $f()$... – Steven Stadnicki Jun 04 '14 at 19:57
  • There is no room for a proof in the comments, since it long. Ask a separate question. – Moishe Kohan Jun 04 '14 at 19:58
  • Hm, i would say it can be shown as a corollary of Jordan–Schoenflies theorem (of which i do not know a proof). – Alexey Jun 04 '14 at 20:10
  • 1
    I'd agree w/ @Alexey. Add a curve from $(1,1)$ to $(0,0)$ to extend $f$ to a closed curve $\tilde{f}$ that $(0,1)$ is inside and $(1,0)$ is outside. Find a homeomorphism $\Phi:\mathbb{R}^2\rightarrow\mathbb{R}^2$ of the plane that maps $\tilde{f}$ to the unit circle (by the Jordan-Schoenflies theorem). Draw a straight segment connecting the images of $(0,1)$ and $(1,0)$ (this crosses the unit circle exactly once). The pre-image of this segment (i.e. its image under $\Phi^{-1}$) is the desired curve $g$. (Right?) – mjqxxxx Jun 04 '14 at 20:51
  • 2
    @mjqxxxx: No, that does not work: the straight line segment might not be contained in $\Phi([0,1]^2)$ (since you do not know what the image of the rest of the boundary of the unit square looks like). One needs more work to get a real proof. – Moishe Kohan Jun 04 '14 at 21:30
  • @studiosus: You're right! I didn't notice that. And there's no reason the image of the interior of the unit square under $\Phi$ should be at all nice. Maybe that's the wrinkle here. – mjqxxxx Jun 04 '14 at 21:48
  • This is a bit of a stab in the dark, but can there even be such a g if f is a space-filling curve? – Christopher Creutzig Jun 04 '14 at 21:57
  • @ChristopherCreutzig: Both maps f and g are 1-1, hence cannot be space-filling. – Moishe Kohan Jun 04 '14 at 22:00
  • 1
    Here is a stronger (straightening) theorem: Let $S$ be a surface with some fixed triangulation (say, a triangulated plane) and $h: G\to S$ a topological embedding of a finite graph. Then $h$ is isotopic to a piecewise-linear embedding $h': G\to S$. This reduces problems of the above type to piecewise-linear ones which are easily solvable. One can give, however, a more direct solution using only Schoenflies and a bit of algebraic topology. – Moishe Kohan Jun 04 '14 at 22:03
  • Yeah, my counterexamples all actually fail continuity, I hadn't been paying close enough attention (they had 'accumulation lines', Topologist's Sine-curve style, but of course that breaks the assumption). I'll delete this non-false theorem in a bit and ask my example as a proper question, since the proof sounds really interesting to me! – Steven Stadnicki Jun 04 '14 at 22:04
  • @studiosus (You can find that question at http://math.stackexchange.com/questions/821025/is-this-variant-of-the-jordan-curve-theorem-true if you want to answer the question properly there!) – Steven Stadnicki Jun 04 '14 at 23:22
5

The following is obviously false, but it is actually true, as shown in the Wikipedia article about Vitali sets.

The exists a countable collection $\left\{V_n\right\}$ of subsets of the unit circle such that:

  1. Any two distinct $V_n$ are disjoint.
  2. Any $V_n$ can be obtained from any other by a rotation.
  3. The union of all the $V_n$ is the whole circle.

All the $V_n$ must have, from property two, the same "size" (for any reasonable definition of "size"), but if the above fact was true, the sum of their (equal) sizes would be the size of the circle (positive, but finite). But if the size was zero, the the sum should be zero, and if the size was positive, the sum should be infinite.

A consequence of this is that the following is false (although we all would like it to be true):

There exists a function $\mu$ that, given a bounded subset of $\mathbb{R}$, tells you its "size". Precisely:

  1. If $A\subset\mathbb{R}$ is bounded, then $\mu\left(A\right) \in \left[0,\infty\right[$.
  2. If $\left\{A_n\right\}_{n\in\mathbb{N}}$ is a sequence of bounded disjoint subsets of $\mathbb{R}$ (that is, $A_n \cap A_m=\emptyset$ whenever $n\neq m$) with bounded union (that is, $\bigcup A_n$ is bounded), then $\sum\mu\left(A_n\right)=\mu\left(\bigcup A_n\right)$.
  3. If $A$ is bounded, $x$ is a real number, and we define $A+x=\left\{a+x:a\in A\right\}$, then $\mu\left(A+x\right) = \mu\left(A\right)$.
  4. $\mu\left(\left[0,1\right]\right) = 1$

Indeed, rewriting the first fact exchanging the circle by the half-open interval $\left[0,1\right[$ and exchanging rotations for cyclic shifts "mod $1$", we realize that if $\mu$ satisfies the first three conditions above, then $\mu\left(A\right)=0$ for all $A$.

fonini
  • 2,517
  • 1
  • 14
  • 43
  • If you consider a **finite** number of sets $V_n$, and take a limit it does seem plausible, not quite obviously false. – Mark Hurd Jul 28 '15 at 07:35
5

The Birthday Paradox

If 30 people are randomly selected, and they have birthdays that are independently, (identically) uniformly distributed over the calendar year, then the probability that two (or more) of them have the same birthday is approximately $\frac{1}{12}$.

Scott
  • 1,015
  • 1
  • 13
  • 27
4

A linear order can be uniquely (up to isomorphism) reconstructed from the set of order types of its proper initial segments.


Update: Even if we know the cardinality of the linear order, and know that it does not have a maximal element, this "theorem" still does not hold.

Vladimir Reshetnikov
  • 45,303
  • 7
  • 151
  • 282
4

All infinities are of the same size.

But Cantor's theorem shows otherwise.

Alex Vong
  • 1,578
  • 11
  • 21
4

Here are some "obvious" statements to which Richard's paradox can apply to:

  1. For a given predicate $P$, there exists a set $S$ of $x$ for which $P(x)$ is true. (Russell's paradox)
  2. The set of integers and the set of real numbers are the same infinite size. (Cantor's diagonal argument.)
  3. There exists a formalization of arithmetic in which all true statements are exactly those which are provable. (Gödel's theorem(s))
  4. There exists a computer program (Turing machine) that can effectively determine if any other computer program doesn't halt. (Halting problem)
Matt
  • 694
  • 5
  • 14
  • 1
    [I already mentioned #1](http://math.stackexchange.com/a/820785/25554). – MJD Jun 04 '14 at 19:53
  • @MJD Yes and I +1'd it :) Your harping is well-justified. – Matt Jun 04 '14 at 19:54
  • 2
    I think #4 is not merely not obviously true, but obviously false, because if it were true, then we wouldn't have to actually run our computer programs to find out what they did, instead we could use this hypothetical procedure to tell us, without running them. But then what would be the point of running the programs at all? The universe just doesn't work that way; if you want to find out what happens, then, at least some of the time, you just have to try it and see. – MJD Jun 04 '14 at 19:55
  • Also #2 is already mentioned in my answer. – Asaf Karagila Jun 04 '14 at 19:56
  • @MJD My apologies, I just corrected a typo in #4 in which I replaced "halts" with "doesn't halt" at the end of the sentence. – Matt Jun 04 '14 at 22:06
  • 1
    It doesn't matter; my comment is the same either way. I have a blog post drafted that claims that Rice's theorem is the *most* obvious theorem in mathematics, and the undecidability of the halting problem is a special case of that. – MJD Jun 04 '14 at 23:59
  • 1
    @MJD I don't see the essence of your point. Part of the halting problem is if a program doesn't halt, how will we know that? We *can't* just "try it and see"--how long do we wait before deciding it doesn't halt? This is the essential problem. This kind of statement may have been more "obvious" at the beginning of the 20th Century when automation seemed to be the name of the game, also reflected in the goals of Russell and Whitehead in formalizing mathematics. It's great that it is obviously false to you now, but I would say it's *because* of results like this that we are now so enlightened. – Matt Jun 05 '14 at 17:15
  • Let me put it another way. If the halting theorem were false, then there would be a method to tell, rather quickly, what the outcome of any computation would be, even if that outcome was in the distant or infinite future. But we *know* that foretelling the distant future is impossible; it is in the same category of fantasy as leprechauns and unicorns. So why should we be surprised that there is not, in fact, any way to tell now what a Turing machine will do in the eventual future? You claim that it's obvious (but false) that we could correctly predict the infinitely-far future. Why? – MJD Jun 06 '14 at 13:40
  • 1
    This is not about general predictions of the future; this has to do with time only insofar as countable infinity can be mapped to discrete Turing machine steps that take place in time. E.g. A Turing machine that halts when it finds $a,b,c,n\in\mathbb{Z}_+:a^{n+2}+b^{n+2}=c^{n+2}$ we know doesn't halt thanks to Wiles's proof. If the halting theorem were false, then a "theorem-proving machine" based on a formalization of any area of mathematics would spit out all true statements eventually. This was "obvious" to Russell, Whitehead, and even Hilbert until the 1930's thanks to Gödel, Turing et al. – Matt Jun 06 '14 at 19:42
  • 1
    @MJD If the halting theorem were false, it doesn't imply that *we* would be able to tell the outcome of any computation. It merely would imply that an algorithm exists. There is still a substantial gap between the existence of such an algorithm $U$, and both (1) our physical ability to run $U$ (it may require more memory than the number of atoms known to exist) and (2) more importantly our *knowledge* that a given algorithm is in fact $U$. (1) and (2) do not automatically follow from the halting theorem's falsity. The reasons for it being true are more subtle than you are framing it. – Matt Jun 06 '14 at 19:53
  • @MJD I agree with Matt. Even if the halting theorem were false, it doesn't mean that the algorithm would be at all practical, and of course reason demands that it be wildly impractical. As Matt points out, a theorem proving machine can spit out every true statement if the halting theorem was false; since it is true, such a program will only spit out provable statements, and cannot tell you that a given theorem is unprovable unless you wait forever. But even that sounds great - why don't we run theorem proving machines to spit out all provable statements and put mathematicians out of business? – Mario Carneiro Jun 11 '14 at 00:10
4

There are a lot of examples in (extremal) graph theory, where an obvious argument shows that a statement is true, except that there are a number of small counterexamples which are easy to overlook.

Consider the following statement: Let $G$ be a graph with $n$ vertices and the largest number of edges subject to the condition that $G$ does not contain a pair of disjoint edges (i.e. $K_2 + K_2$). Then $G$ is a star (i.e. $K_{1,n-1}$).

This is obviously true, if you think about it for a moment. But for $n=3$, a better solution is to take $G = C_3$. And for $n=4$, taking $C_3$ plus an isolated vertex is just as good as taking $K_{1,3}$.

Jakub Konieczny
  • 12,084
  • 1
  • 30
  • 71
3

A subgroup of a finitely generated group may not be finitely generated and there are up to isomorphism at most two $pq$ groups, where $p$ and $q$ are prime.

To strengthen your example, $\sum_{p \ prime} 1/p$ diverges.

The continuum hypothesis also seems like it has to have in answer in ZFC, which it doesn't.

On another page, mathematicians thought that cyclotomic fields "obviously satisfy the unique factorization theorem" leading to some false proof attempts to Fermat's last theorem.

Next, one might think that the "angle trisection" is possible or that any set of analytic functions $\{f_\alpha\}$, such that for every $z \in \mathbb C$, the set $\{f_\alpha(z)\}$ is countable, itself has to be countable.

These are just some random examples that came to mind and since the term "obvious" is subjective, you may very well disagree with items on my list. I guess it heavily depends on your mathematical background.

Stefan Mesken
  • 16,151
  • 3
  • 25
  • 47
  • 5
    I disagree that there is *anything* intuitively obvious about the continuum hypothesis, and I find even less that is intuitively obvious about what can or cannot be proved in ZFC, which is so complex that you have to study really hard just to understand what the axioms are saying. – MJD Jun 04 '14 at 18:31
3

The following is a very well-known example, though probably slightly outside of the world of mathematics, rather physics. A great many people would 'intuitively' consider the following to be true:

The heavier the object, the faster it falls down.

In fact, the story goes that this was supposed to be common knowledge until Galileo Galilei disproved it (as the story goes, by dropping two balls of the tower in Pisa, which never happened though).

One of the first physics classes many people have (I'm talking primary school here) aims at showing this theorem is false, and actually everything falls with the same acceleration (ignoring resistance by air) regardless of weight.

Yellow
  • 157
  • 1
  • 9
  • 1
    Everything falls with the same *acceleration* not at the same speed... big difference. – Dan Jun 05 '14 at 08:29
  • 3
    Aristotle's claim was actually that objects fall with speed *proportional to their weight*, which is much more specific, and so much less obvious-seeming. It is incredible that Western civilization swallowed this whopper for as long as it did. – MJD Jun 05 '14 at 11:47
  • 1
    @MJD Universally people tend to believe things according to popularity rather than evidence or experience. This is as true today as it was in the past, and studies show that (counterintuitively) well-educated people are better at doing it than poorly-educated people. – AndrewC Jun 05 '14 at 16:22
  • I make fun of this idea in my lessons when I've checked several students answers and pick the mode answer, and announce "This one must be right, because as we all know, democracy is the route to all truth!" – AndrewC Jun 05 '14 at 16:22
  • 2
    But a gas balloon is lighter (relative to its volume) than eg a rock and it even rises instead of falling down. – Robert Jun 08 '14 at 20:57
3

If you start thinking about the rigidity of thin shells in $\mathbb{R}^3$, you quickly encounter a slew of counterintuitive results.

For instance, it is obvious that a spherical shell is ($C^2$) rigid, and this is in fact true. A smooth, closed, compact surface with everywhere positive Gaussian curvature is likewise rigid. One might imagine these results generalize to

  • Any closed surface;
  • Any closed surface with positive Gaussian curvature everywhere but at finitely many points;
  • Any surface with boundary with everywhere positive Gaussian curvature;

and all of these are false.

Moreover, after thinking about reflections, or poking and prodding at a ping-pong ball, it is intuitively obvious that a spherical shell is not $C^0$ rigid. But you can't really "see" any difference between a $C^1$ and a $C^2$ deformation of the sphere, so surely the sphere is $C^1$ rigid? Far from it -- given any arbitrary closed surface that is topologically a sphere, and distance $\epsilon$, it is possible to $C^1$-isometrically embed a sphere $\epsilon$-close to the target surface!

user7530
  • 45,846
  • 11
  • 84
  • 142
3

Here is a proposition, when put in layman's terms, an average person would claim to be true.

Every subset of real numbers has a measure.

How can this be false, when you mark a region, say in two dimensions, of course, it has an area?! Unless you constructed a Vitali set at some point, we tend to think that the concept of length/area/volume should extend to all possible subsets.

Here is another such false proposition.

Axiom of Determinacy

If we are playing a two-player infinite game where we create a real number in $[0,1]$ by choosing decimal digits in turns and one of us tries to land the resulting number in a pre-determined payoff set that is known to both of us and the other tries to avoid it, how could it be that there is a game where neither of us have a winning strategy? We both have complete information about the payoff set what numbers to avoid and what numbers to hit, one of us should be able to come up with a strategy. Well, unfortunately no.

Both of these propositions are inconsistent with the axiom of choice, with which you can construct the counterexamples that will not "nicely behave".

Fact: The latter proposition implies the former. (in ZF, with which AD is believed to be consistent).

Burak
  • 3,516
  • 3
  • 15
  • 26
  • It seems a bit of a stretch to call AD "false." AC is not on any higher epistemological plane. – Kevin Arlin Jun 05 '14 at 20:51
  • How would you put "every set of real numbers has a measure" in layman's terms, so that an average person would claim it to be true? Be sure to include countable additivity in your layman's formulation, since that's needed to make the proposition false. – bof Jun 05 '14 at 22:23
  • @bof: The concept of measure originates from our intuitive notion of length/area/volume. I doubt that an average person would object to that. Pick a unit square and color a subset of it, it should have some "area" comparable to areas of basic shapes. Or, ask this, if I throw a dart to this square, do you think that there is a well-defined probability that I hit the colored region? As for the countable additivity, doesn't our intuition about "area" include this? If I give you disjoint pieces that you can label by naturals, of course the area of the union is the sum of the areas? – Burak Jun 05 '14 at 22:48
  • @KevinCarlson No stretch, but common usage. "False" (without qualifiers, as in "false in this or that model") is shorthand for "false under the standard interpretation", which in this case means under the standard set of axioms, namely $\mathsf{ZFC}$, including choice. – Andrés E. Caicedo Jun 06 '14 at 02:58
3

It's not exactly a theorem, but it fools every math newcomer:

$e = \lim_{n\to\infty} \left(1+\frac{1}{n}\right)^n$

$(1 + 1/\infty)$ is $1$, obviously. And 1 to the power of $\infty$ is obviously still 1.

Nope, it's 2.718...

Cephalopod
  • 287
  • 1
  • 7
3

$\textbf{Counter-Intuitive Example}$

$$\ \ \textbf{D}_v f(\textbf{a}) = 0, \forall \textbf{v},a \not \Rightarrow f \ \ \text{continuous}.$$

Mr.Fry
  • 4,913
  • 2
  • 16
  • 26
3

Geometry proofs done informally by drawing figures on the blackboard. You then bypass the axioms of Euclidian geometry, you pretend that you don't need to invoke them as the figures drawn seem sufficient. However, in Earth's gravity Euclidean geometry is only an approximation.

Count Iblis
  • 10,078
  • 2
  • 20
  • 43
3

Infinitely many terms always have a sum equal to infinity.

Harshal Gajjar
  • 1,634
  • 4
  • 21
  • 33
2

Another example of 'obvious' being not true is Bus Waiting Time Paradox.

If the mean time between two consecutive buses arriving to a bus station is $M$, one should expect that the mean time you have to be waiting in the station before the next bus arrives is also M. But this is not true; and depending of the particular bus arrival time distribution you'll have to wait a time $M'\geqslant M$

Perspectiva8
  • 348
  • 1
  • 15
2

Someone else mentioned "there are more rational numbers than integers". Along the same lines, I had a hard time accepting that

There are more integers than there are real numbers between 0 and 1

is false. I mean, I get it now, but it seemed intuitively very wrong to me before I studied transfinite numbers.

Old Pro
  • 354
  • 2
  • 14
1

Perhaps this isn't quite what you were looking for, but it's still fun! How about a proof that is obviously incorrect but (to newcomers) it is difficult to figure out what is wrong.

Let $x = y$. Then $$x^2 = xy$$ $$x^2-y^2=xy-y^2$$ $$(x+y)(x-y)=y(x-y)$$ $$x+y = y$$ $$2y=y$$ $$2=1$$

Auden Young
  • 1,762
  • 1
  • 15
  • 30
1

An "obviously true" theorem:

If take a 3d object $U$ and chop it into finitely many pieces, any object $V$ I rearrange those pieces into will have the same volume as the object $U$ I started with.

But in fact, the Banach-Tarski paradox tells us this isn't true -- if we construct our finitely-many subsets of $U$ "weirdly" enough, we can actually build a $V$ with any volume we'd like.

tyo
  • 525
  • 2
  • 12
1

Any continuous function is differentiable at least somewhere, right?

False, the Wierstrass function is a famous counterexample

https://en.wikipedia.org/wiki/Weierstrass_function

It is continous everywhere, yet it is differentiable nowhere.

Some Guy
  • 2,620
  • 3
  • 13
  • 28
1

Here is one thing commonly thought to be true but is quite horribly wrong on many accounts:

There is a notion of mathematics where we can say things are 
"actually true" or "actually false".  

An example of making this error: the OP. Other examples: the many responses.

There are several reasons why this is wrong. First, in the system most mathematicians assume when not being explicit, we have no standard model (we have no models within that system because any model would show the system consistent which we know we cannot show in that system through blah blah Godel blah.. I know you don't want details, just explaining what I'm getting at). Truth and falsity are semantic - they exist in models and so without one, we don't make claims of truth or falsity.

But also, mathematics is not "the system most mathematicians assume when not being explicit" - it is formalisation in general. There are many systems seriously investigated by mathematicians that make numerous "counterintuitive" derivations. For instance, these are all obvious and wrong in different systems:

  • A statement cannot be both true and false. (In paraconsistent logics, statements can be both true and false and the system does not collapse to trivial - in fact, a number of dialetheists argue this is a much more accurate logical system for real world reasoning).
  • There are discontinuous total functions. (In a number of constructive systems it is not possible to prove the existence of discontinuous total functions. Some are even strong enough to prove that all total functions are continuous.)
  • Every infinite set A has the same cardinality as AxA. (This is not necessarily true in systems without the axiom of choice. Famously, Tarski tried to publish his result on this implication and was rejected by both Frechet and Lebesgue. Frechet thought the paper was obvious and well-known and had no mathematical merit. Lebesgue thought both the axiom of choice and the implication from it were both wrong, so the paper had no mathematical merit.)

I only bring these examples up not as answers to the OP but simply to illustrate my real answer that the question itself demonstrates an extremely common assumption in mathematics that is in fact wrong.

EDIT

This is an area that I think is often a place of common misunderstanding, and discussion in the comments makes it clear I should elaborate. Modern mathematics separates out the domains we make statements on into syntax and semantics.

Syntax

The syntax is the theory - the formal language, axioms specified as sentences in the formal language, and some metalogical rules of inference. In the syntax, we talk about sentences, propositions, terms, derivations, and proofs. It is a place of symbol manipulation.

Semantics

The semantics is the model - it is the meaning we ascribe to the statements of the theory. An interpretation of a theory is a model that assigns to each formula of the theory a meaning value - typically truth. Truth is semantic and is specific to a model.

The "problem"

A model is a consistent interpretation of the truth meaning of a theory. If a theory has a model, it has almost trivially been shown to be consistent. But... it is well known that a theory strong enough to express the Gödel diagonalisation can never prove it's own consistency. For these theories, we will never have a model and cannot make statements about the meaning of any formula.

In these theories, it is wrong to talk about truth or falsity. We don't have a model giving meaning to that. We will never have a model.

That's not really a problem. For centuries, mathematicians had loosely combined derivation and truth and had mostly discussed them as one thing. Derivation and proof were seen as the important part of mathematics and formalization. You still have that.

Also, it is perfectly meaningful to derive results that say "if this theory is consistent and has a model, then...". Model theory has been doing that for nearly a century.

What about truth predicates?

But people seem to want more. They want to talk about truth, as that is a form of meaning that holds a special place. They often go to great lengths to try to continue to assign truth and falsity. One common approach is to form truth predicates - predicates in the syntax that have the property that asserting the predicate on a formula corresponds to asserting the validity of the statement (that it is true in all models).

Note the switch - a truth predicate is syntactical. We still aren't talking about true or false here - the context of their use is still whether statements including the predicate "are derivable" or "obtain". Theories may have multiple models - most theories are not categorical just from things like Löwenheim–Skolem, so predicates cannot talk about truth. They can talk about validity - and that's really what is going on here - but even that is extremely problematic.

Incomplete theories cannot actually derive anything about validity on the total theory. And actually, this is where Tarski's theorem on nondefinability comes in and it is shown that such a predicate doesn't actually exist. So others keep at it with a hierarchy and reflection extensions of the base theory, seeking out some approximation of a fixed point for validity.

But this doesn't actually buy anything to do with insight into truth. It cannot. There is nothing you can do to reach truth because you cannot know if the theory is consistent or not and whether truth exists. And no attempts to reach beyond derivability actually give a predicate that can be used and say "this is true". The predicate is only useful to say "this is provable".

But there are already provability predicates, and that investigation is much more profitable. Truth predicates are a voiceless oracles. They do not help anyone make assertions on truth. They are simply reformulations of "if we knew that X was consistent, and we had some platonic sight that could see the truth values in all models, and we could collate the infinite possibilities and see the validities forever hidden, then this predicate applied to this class of statements would agree with those assertions that are valid". But if we had that supernatural sight, we could more easily just say "hey, that's true in that model - and that's false over there." Without that, we can use the predicate to say "truth is preserved in this derivation". Which doesn't add anything.

A truth predicate doesn't talk about truth. It is irrelevant to the point.

So...

So.. life goes on. My whole point in posting this answer was to illustrate that the initial question was making a common obvious assumption that is actually wrong. You should not talk about truth in the commonly used ambient theory - just talk about what is provable and you are fine. If you want to talk about truth, ensure you specify the ambient theory and it is one where such discussions are meaningful. Or talk about conditional models as model theorists do.

It may not be intellectually satisfying to some people. Clearly, as of writing this, my answer has received 3 downvotes and two upvotes, so it doesn't sit right with some anonymous readers of a math web site. But there is nothing controversial about the point. It has been known for almost 100 years and it is still a common mistake.

ex0du5
  • 1,338
  • 8
  • 14
  • I think that the third one is neither obviously true nor obviously false. – Asaf Karagila Jun 04 '14 at 22:54
  • @AsafKaragila: I do too. I put it there because there is a famous anecdote about two mathematicians who believed it obvious, but on both sides. It was more to illustrate the point that many make the category mistake of thinking theorems "true or false" instead of "obtains in ambient system S". Truth or falsity, indeed even validity, are not really applicable when your ambient system cannot provide a semantics on which to make those claims. – ex0du5 Jun 04 '14 at 22:59
  • 3
    Your third paragraph, in its most precise form, is Tarski's undefinability theorem. But being aware of what it is, I still disagree with the preceding two paragraphs. Relative to another system, you can talk about truth, for example arithmetical truth is definable in ZFC (but not in PA). Truth in the set theoretic universe is not definable in ZFC, but so what? – Burak Jun 04 '14 at 23:02
  • If you believe that there is a set theoretic universe and first order logic captures its truth (meaning that axioms are true [whatever this means]), then we can interpret being provable/disprovable as being true/false. Why do we have to be able to define (formal) truth in the universe which we presume to exist? If talking about truth and falsity bothers you, you can re-read the question and all the answers (that you seem to claim are faulty) as "provably true" and "provably false". – Burak Jun 04 '14 at 23:04
  • One last quick remark: Formal truth in the set-theoretic universe is actually definable, but not uniformly. What this means is that if you put a bound on the complexity of your formulas (say in the Levy hierarchy), then you can define the truth for those formulas (using a single formula having greater complexity). Not being able to do this at a single stroke (with a single formula) should not make you think that mathematical truth does not make sense nor actually true/false are meaningless. – Burak Jun 04 '14 at 23:14
  • @Burak: My comment gave two reasons why those preceding paragraphs were wrong. Yes, the first was the standard "consistency proven in the same theory for theories strong enough for PA" to point out that simply assuming the standard ambient theory was insufficient to make those claims. The second reason, though, was pointing out that if you don't assume the standard ambient, then the notion of what is true or false changes dramatically based on ambient theory. Neither of those two statements are controversial, and they cover what I was trying to show. – ex0du5 Jun 04 '14 at 23:16
  • @ex0du5: I see what you are trying to show. On the other hand, change in the background theory effecting the formal truth does not make the question meaningless, nor any of the answers. You can simply assume that the OP's question takes place in ZFC as the background theory (or some theory where he can formalize objects of his theorems). It might be my misunderstanding of your answer but I interpreted what you've written as an extremely-skeptic approach using incompleteness/undefinability phenomena to reject any kind of semantic notion. – Burak Jun 04 '14 at 23:24
  • @Burak: There are many ways to start a programme to "fix this up". But it is important to be clear about what those programmes are attempting. Provability and Truth are separate and have been clearly delineated in metamathematical investigations since before Godel. We can give a modality to provability and talk of accessible relations, but the terms to use here are "derivable" or "obtainable". Going your other way - truth definitions are not truth. In both discussions, there is a category error mistaking syntax for semantics. (cont.)... – ex0du5 Jun 04 '14 at 23:32
  • @Burak: One way to look at my response is that I am pointing out "it is a common assumption that you can talk about semantics in ZFC in the way mathematicians have developed over the past century" as you clearly can't. You can try to look away from semantics being important, you can take the existence of a semantics as a belief, you can substitute syntactic definitions, but nothing gives that semantics in the way mathematics has developed. In fact, those points were precisely some of the reasons that caused the need to be precise about truth and semantics. – ex0du5 Jun 04 '14 at 23:36
  • @ex0du5: I see. So you want to point out the difference between formalization of truth and "actual truth" (whatever that means)? Am I right? If so, I agree that the distinction is important to keep in mind but since there is no way of doing metamathematics without formalizing these concepts, discussing the difference seems like philosophical issue. The same issue applies to natural numbers, as well. Is it the case that our axiom systems are arithmetically sound, meaning that what they prove is "actually" true in "the" natural numbers? There is nothing mathematical to do against this question. – Burak Jun 04 '14 at 23:48
  • @Burak: I'm not saying anything against formalization. Quite the opposite, I am saying there is a clear formal description of truth that has been refined and applied to the great benefit of mathematics. There is no philosophy in my answer. Truth is "truth in a given model of a given theory". Assuming another meaning to truth has been a no-no for a century, and there are well-known reasons where that mistake led mathematicians astray. I truly am answering the original question in giving an actual "obvious" belief that is, in fact, wrong. I am not trying to be subtle. – ex0du5 Jun 05 '14 at 00:22
  • @ex0du5: If we agree to use the Tarskian notion of truth, then I must repeat my first three comments. You are saying in your post that there is no notion of mathematics where we can say "actually true". This is Tarski's undefinability, nothing more. What I am saying is that if you are willing to sacrifice uniformity, then there is a definable notion of "actually true". I can give you an inductive description of true sentences, say in set theory, having at most n unbounded quantifiers provided that you fix n. (cont'd) – Burak Jun 05 '14 at 00:30
  • @ex0du5: To clear our miscommunication, what exactly is the statement (in a *mathematical* form) that you claim is thought to be obviously true but is wrong on many accounts? – Burak Jun 05 '14 at 01:13
  • Gödel's incompleteness theorem really drives me into weird philosophic discussions after a beer or two, as it states that the rules of math do not base on laws of nature and are only tools 'made up' by us: "Any effectively generated theory capable of expressing elementary arithmetic cannot be both consistent and complete." – aRestless Jun 05 '14 at 21:57
  • @Burak: I expanded my answer. I hope it clears up why partial predicates can never speak the ontology of truth and how syntactic constructs do nothing to address the point of the inaccessible semantics of theories strong enough for the Godel construction. If not, I don't know what to say as this is very well-known and hardly controversial, though I understand it leaves some people with an existential angst. – ex0du5 Jun 05 '14 at 23:01
  • @aRestless: I think your comment on the laws of nature expresses an important part of why this leaves so many with such a strong impression. In nature, if we talk completely operationally, we can know the answer to a semantic query simply by "looking" (performing the operation or experiment). All truth is reachable. So why not math? Particularly something so simple as arithmetic? – ex0du5 Jun 05 '14 at 23:22
  • @ex0du5: Your expansion of the post at least cleared up where my point of view differs than yours. We can cook up a (formalized-)truth predicate that would tell us whether a (formalized-)sentence is true in a model or not. You are saying that these predicates don't talk about "truth". I tend to think that they do, since this kind of formalization (embedding semantic concepts into syntax) of truth seems like the only way to talk about truth mathematically (because this is how mathematics has been done for a long time, syntactically, in a formal system). – Burak Jun 05 '14 at 23:25
  • @ex0du5: A side comment about your "the problem" paragraph. Even in the case that our theory proved its own consistency (or equivalently that it has a set-model), since you are denying that truth predicates talk about truth, you wouldn't be able to talk about truth of a sentence. Otherwise, all your problems could be solved via an axiom asserting the existence of a model of your theory. Asides from that, I am not sure that it is "well-known" that truth predicates (or any formalization of semantic concepts in model theory) are in fact meaningless. – Burak Jun 05 '14 at 23:39
  • @ex0du5 You wrote: "a theory strong enough to express the Gödel diagonalisation can never prove it's own consistency". Is this statement "actually true"? – Vladimir Reshetnikov May 18 '16 at 01:34
  • @VladimirReshetnikov I don't know. But I do know it is provable in those theories. – ex0du5 May 18 '16 at 04:21
  • @ex0du5 You wrote: "it is provable in those theories". Apparently, this is a statement universally quantified over certain theories. Is this statement actually true? – Vladimir Reshetnikov May 18 '16 at 05:25
  • @ex0du5 I suppose you did not manually checked provability in all of the infinite multitude of theories you are quantifying over. How can you say "I do know it is provable in them" without ascribing any truth value to the statement expressing provability in a theory? To know something at the very least means you believe it is true, doesn't it? – Vladimir Reshetnikov May 18 '16 at 23:21
  • @VladimirReshetnikov I'm sorry that my response was light hearted. I took your initial question the same. Being honest, though, I think the only foundations that earnestly approach this question are radical constructivists somewhere between predicativism and ultrafinitism, where such quantification is invalid. I personally have a lot of affinity for the computational foundations in those schools and their strong focus on semantics, as I think that is the only approach to meaningfulness that can work. I have tried to avoid that above and stick to stuff widely accepted, but maybe it can't avoid. – ex0du5 May 18 '16 at 23:32
1

For me a nice example of all of "evidence" suggesting it was true is

$$\pi(x) < \operatorname{li}(x)$$

until Skewes showed that $\pi(x) - \operatorname{li}(x)$ changes sign infinitely often

MJD
  • 62,206
  • 36
  • 276
  • 489
scibuff
  • 651
  • 5
  • 13
  • 8
    The question asks for obvious but false statements. Are you seriously claiming that $\pi(x) < \operatorname{li}(x)$ is obvious? – MJD Jun 05 '14 at 21:32
  • 12
    hard to tell whether that is supposed to be obvious... What are $\pi(x)$ and $\operatorname{li}(x)$? – example Jun 05 '14 at 22:03
  • $\pi(x)$ is the [prime counting function](https://en.wikipedia.org/wiki/Prime-counting_function), $\mathrm{li}(x)$ is the logarithmic integral. Ramanujan asserted the inequality. Littlewood proved in 1914 the sign changed infinitely often and [Skewes gave an upper bound](https://en.wikipedia.org/wiki/Skewes%27s_number) for the first change. – Henry Feb 07 '21 at 18:45
1

The following statement is wrong:

The inner angles of a triangle always sum to 180 degrees.

While it sounds plausible that the sum of the angles is a constant, it is actually a property of the space. In Euclidean space the inner angles of a triangle always sum to 180 degrees.

Robert
  • 195
  • 7
  • But it is fairly obvious and it is true. – bjb568 Jun 09 '14 at 01:48
  • @bjb568 I think Robert may be thinking of non-planar triangles. – MJD Jun 09 '14 at 05:16
  • @MJD Well, that makes more sense, but then it isn't obvious… – bjb568 Jun 09 '14 at 06:05
  • Yes, it seems obvious that the sum of the angles is a constant. But actually this is only always true for planar triangles. – Robert Jun 09 '14 at 08:38
  • 2
    It doesn't seem obvious to me that the angles of a plane triangle always have the same sum. In fact, it seems astounding, and I'm sure you could find other places on this web site where other people said it seemed astounding. – MJD Jun 09 '14 at 14:14
  • 2
    For example, [this highly-voted answer](http://math.stackexchange.com/a/323359/25554) begins “I found it completely amazing that the angles in a triangle always added up to 180 degrees”. – MJD Jun 09 '14 at 21:22
  • I see that the correctness of the claim is not immediately obvious. Also it's rather nitpicky to point out that its correctness is only proven in euclidean geometry. But I always thought it was at least easy to see since obviously the exterior angles of any convex n-gon sum to 360+180n degrees. So I was taken aback when I learned about non-euclidean geometries. Now I'm not sure what to do with this answer. Should I edit/delete it or just leave it as it is? – Robert Jun 10 '14 at 01:29
  • Well if we let the word triangle be a 2d, 3 sided, polygon. Then yes because if we increase the lengths of a line segment one must also increase also, think about if this wasn't true it would mean a trinagle could have two obtuse angles because with the property of all inner angles adding up to 180 degress it would require if angle was 91 degress that the others add up to 89 degress which isn't obtuse. I see you point in a special type of space but because in the space we live in works the way it does, an trinangle's inner angle's add up to 180 degress... (I'm going to go on below.) – Binary Freak Jul 23 '14 at 20:31
  • Contiuing... This also because a triangle is property of our space, by your defintion because of expanding and shorting triangle lengths needed to change triangle angles which would make it not a polygon because it would have to be an open shape. – Binary Freak Jul 23 '14 at 20:37
1

If a propositional calculus A contains all theorems of propositional calculus B under detachment and uniform substitution for propositional variables, but B does not contain all of the theorems of A, then one of the shortest single axioms of A is longer than any of the shortest single axioms of B. Or one might more haphazardly say "if propositional calculus A is bigger than propositional calculus B, then one of the shortest single axioms of A is longer than any of the shortest single axioms of B."

Doug Spoonwood
  • 10,720
  • 1
  • 30
  • 50
  • Have you stated the false "theorem", or is this a true counter-intuitive statement? (Clearly I'm not actually intuiting anything :-) ) – Mark Hurd Jun 12 '14 at 16:21
  • @MarkHurd It's a "false theorem". The pure implicational calculus has a 13 letter single axiom, but the single axioms for say BCI are longer. – Doug Spoonwood Jun 12 '14 at 16:31
  • 3
    Good luck finding a layman whose eyes don't glaze at the term "propositional calculus", let alone one who finds something here obvious. – rumtscho Jun 13 '14 at 17:46
  • @rumtscho I don't work in academia. I also only took one logic course in college, and 2 calculus courses. – Doug Spoonwood Jun 13 '14 at 23:21
0

An analytic function with compact support vanishes identically.

  • 9
    That is actually true, as long as we stay in $\mathbb{C}^n$ resp. $\mathbb{R}^n$. It becomes false when one takes a compact analytic manifold, but then it's obvious that it's false. – Daniel Fischer Jun 04 '14 at 19:45
0

Lebesgue once stated that the projections of Borel sets in $\mathbb R ^2$ on to one of its axes are also Borel sets. This fact is actually false, the realization of which is attributed to the short-lived mathematician Mikhail Yakovlevich Suslin.

Unfortunately it is very difficult to find a counterexample. The only one I've ever seen takes a result in descriptive set theory about $\mathbb N ^ {\mathbb N}$ and uses the fact that it's homeomorphic to $\mathbb R$.

jjfunk
  • 11
  • 3
  • 3
    I hope you are misremembering or misunderstanding the arguments you have seen, because $\mathbb N^{\mathbb N}$ is not homeomorphic to $\mathbb R$. For example, one is connected and the other one is not. One is $\sigma$-compact and the other one is not. – Andrés E. Caicedo Jun 06 '14 at 02:54
  • That depends on the topology, but it might be that the argument I'm thinking of used the space of all sequences (both finite and infinite) of natural numbers or developed the homeomorphism with the exclusion of $\mathbb Q$. – jjfunk Jun 06 '14 at 04:04
  • 2
    "That depends on the topology". Obviously. "The space of all sequences (both finite and infinite) of natural numbers." Perhaps, but that is not $\mathbb N^{\mathbb N}$. "With the exclusion of $\mathbb Q$." But then that is not $\mathbb R$. Anyway, a word or two clarifying the imprecision would not be superfluous. – Andrés E. Caicedo Jun 06 '14 at 04:51
  • @jjfunk: Is it possible that the map you are talking about is the homeomorphism between the Baire space and the irrationals given by infinite continued fractions? – Burak Apr 09 '16 at 00:58